Sie sind auf Seite 1von 219

PHY 5347

Homework Set 12 Solutions – Kimel

2. 16.2 V = −, q = −e
dE = − τ 〈 dV 2

dt m dr
If V = ar n , then the Virial theorem tells us
〈T〉 = n 〈V〉
2
In the present case, n = −1, so
2
E = 〈T〉 + 〈V〉 = 1 〈V〉 = − Ze
2 2r

dV = Ze 2
dr r2
Now
dE = − τ 〈 dV 2

dt m dr
gives
d 1 = 2Ze 2 τ
dr rt mr 4 t
or
r 2 dr = −2Ze 2 τdt/m
2 e2
But τ = 3 c3m
, so

r 2 dr = −3Zcτ 3 τt

Integrating both sides gives

r 3 t = r 30 − 9Zcτ 3 τt

b) At this point, for simplicity of notation, I’m going to take c =  = 1. Then from problem
14.21,
1 = 2 e 2 Ze 2  4 m
T 3 n5
We are given
2
r = n a0
Z
2 e2
Where a 0 = 1
me 2
= Bohr radius, and τ = 3 m
2 2 2 4 6
− dn = − Z dr = Z 32 Zτ 2 = Z 3 Z Z 2 em = 2 Zm e 5
dt 2a 0 n dt 2a 0 n r 2a 0 n a0n2 3 3 n
in agreement with the result of problem 14.21.
c) From part b
r 30 − r 3 t
t=
9Zτ 2
n 2f a 0 n 2i a 0
But rt = Z
, r0 = Z

n 2i a 0 3 n 2f a 0 3
Z
− Z n6 − n6
t= = 1 a 30 i 4 2 f
9Zτ 2 9 Z τ
In our present case Z = 1, so
3 n 6i − n 6f
t= 1 1 = 1 e −10 n 6i − n 6f 
9 me 2 2 e2 2 4m
3 m

In these units, (from the particle data book) MeV −1 = 6. 6 × 10 −22 s. e 2 = α = 1/137, and
m = 207 ×. 511MeV.

t= 1 × 6. 6 × 10 −22 s n 6i − n 6f  = 7. 53 × 10 −14 n 6i − n 6f s
4 × 207 ×. 5111/137 5
For the cases desired,

t 1 = 7. 53 × 10 −14 10 6 − 4 6 s = 7. 5 × 10 −8 s

t 2 = 7. 53 × 10 −14 10 6 − 1 6 s = 7. 530 0 × 10 −8 s

Just as a check on working with these units, notice


2
τ = 2 em = 2 1 6. 6 × 10 −22 s = 6. 29 × 10 −24 s
3 3 . 511 × 137
in agreement with what we found before.
PHY 5347
Homework Set 12 Solutions – Kimel

1. 16.1 It’s useful to apply in this case the Virial Theorem, familiar from classical mechanics:

〈T〉 = 1 〈 dV 〉r
2 dr
If V = ar n , then
〈T〉 = n 〈V〉
2
In our case V = 1
2
kr 2 , with k = mω 20 , so n = 2 and

〈T〉 = 〈V〉
Or,

〈 dV 〉 = E
r
dr
We are given
dE = − τ 〈 dV 2

dt m dr
This can be rewritten
dE = − τ kE
dt m
So
τ
E = E 0 e − m kt = E 0 e −τω 0 t = E 0 e −Γt
2

Similarly,

dL τ 〈 1 dV 〉L

= −m r dr
dt
But 1 dV
r dr = mω 20 , so

L = L 0 e −τω 0 t = L 0 e −Γt
2
PHY 5347
Homework Set 11 Solutions – Kimel

2. 14.12
a) From Jackson, Eq (14.38)

n̂ × n̂ − ∞⃗ × ∞̇ 2
dP = e 2
dΩ 4πc ⃗ 5
1 − n̂ ⋅ ∞
Using azimuthal symmetry, we can choose n̂ in the x-z plane.

From the figure


n̂ = cos θẑ + sin θx̂

⃗t ′  = − a ω 0 sin ω 0 t ′ ẑ = −∞ sin ω 0 t ′ ẑ


∞ c

aω 2
∞̇ t ′  = − c 0 cos ω 0 t ′ ẑ = −ω 0 ∞ cos ω 0 t ′ ẑ
⃗ × ∞̊ = 0
Using ∞
⃗ × ∞̇
n̂ × n̂ − ∞
2
= n̂ × ∞̇
2

and
n̂ × ∞̇ = ω 0 ∞ sin θ cos ω 0 t ′
So
2
dP t ′  = e c∞
4
sin 2 θ cos 2 ω 0 t ′
dΩ 4πa 2 1 + ∞ cos θ sin ω 0 t ′  5

Defining φ = ω 0 t ′ ,
e 2 c∞ 4 1 sin 2 θ cos 2 φ
∫0

〈 dP t ′ 〉 = dφ
dΩ 4πa 2 2π 1 + ∞ cos θ sin φ 5
Or, doing the integral,
e 2 c∞ 4 4 + ∞ 2 cos 2 θ
〈 dP 〉 = 2 7/2
sin 2 θ
dΩ 32πa 1 − ∞ cos θ
2 2

4+.05 2 cos 2 θ
c) 7/2
sin 2 θ
1−.05 2 cos 2 θ

0
0.5 1 1.5 2 2.5 3

4+.95 2 cos 2 θ
7/2
sin 2 θ
1−.95 2 cos 2 θ

300

250

200

150

100

50

0
0.5 1 1.5 2 2.5 3
PHY 5347
Homework Set 10 Solutions – Kimel

2. 14.2 Background. In the nonrelativistic approximation the Lienard-Wiechert potenials are



φx⃗, t = e |ret , ⃗ x⃗, t = eβ |ret
A
R R
Let us assume that we observe the radiation close enough to source so R/c << 1 and t ′ ≅ t. Then
in the radiation zone

⃗ =⃗
B ∇×A ⃗ = −n̂ ∂ × A
⃗ = n̂ ∂ × A ⃗ = eβ̇ × n̂
∂R c∂t ′ cR

⃗ =B
E ⃗ × n̂

dPt ⃗
= c RB
2 2
= e β̇ × n̂
2
= e 2 |v̇ |2 sin 2 θ
dΩ 4π 4πc 4πc 3
where θ is the angle between n̂ and β̇ (assuming here the particle is moving linearly)
2
Pt = 2e 3 |v̇ |2
3c
Let the time-average be defined by
τ
〈ft〉 ≡ 1τ ∫ 0 ftdt
Then
dPt e 2 sin 2 θ |v̇ |2
=
dΩ 4πc 3

2
〈Pt〉 = 2e 3 |v̇ |2
3c
a) Suppose ⃗
xt = ẑ a cos ω 0 t. Then v̇ = d2z
dt 2
= −aω 20 cos ω 0 t
τ
|v̇ |2 = aω 20  2 1τ ∫ 0 cos 2 ω 0 tdt = aω 20  2 /2
So
dPt e 2 aω 2  2 sin 2 θ
= 0
dΩ 8πc 3

2
〈Pt〉 = e 3 aω 20 
2
3c
b) Suppose ⃗
xt = Rî cos ω 0 t + sin ω 0 t. Then
v̇ t = −Rω 20 î cos ω 0 t + sin ω 0 t

î k̂
n̂ × v̇ = sin θ cos φ sin θ sin φ cos θ
−Rω 20 cos ω 0 t −Rω sin ω 0 t 0

dPt 2
= e 3 Rω 20  2 cos 2 θ sin 2 ω 0 t + cos 2 ω 0 t + sin 2 θ sin 2 ω 0 t + φ 
dΩ 4πc

e 2 Rω 2  2 1 + cos θ
2
dPt
= 0
dΩ 4πc 3 2

e 2 Rω 2  2 2π ∫ 1 1 + x  dx = 2e 2 Rω 2  2
2
〈Pt〉 = 0 0
4πc 3 −1 2 3c 3
PHY 5347
Homework Set 9 Solutions – Kimel

3. 12.14
a) We are given

L = − 1 ∂ α A β ∂ α A β − 1c J α A α

which can be rewritten

L = − 1 ∂ β A α ∂ β A α − 1c J α A α

Using the Euler-Lagrange equations of motion,

∂β ∂L − ∂L = 0
∂∂ β A α  ∂A α

Noting
∂L = − 1 ∂βAα
∂∂ β A α  4π

∂L = − 1 J α
∂A α c

The Euler-Lagrange equations of motion are

∂ β ∂ β A α  = ∂ β ∂ β A α  = 4π
c J
α

or

∂ β ∂ β A α − ∂ α A β + ∂ α A β  = ∂ β F βα + ∂ β ∂ α A β = 4π
c J
α

If we assume the Lorentz gauge, ∂ β A β = 0, then the above reduces to

∂ β F βα = 4π
c J
α

Maxwell’s equations, given by Eq. (11.141).


b) Eq. 12. 85 gives

− 1 F F αβ  − 1 J α A α
16π αβ c
The term in parentheses can be written
F αβ F αβ = 2∂ α A β ∂ α A β − 2∂ α A β ∂ β A α  + 2A β ∂ β ∂ α A α

The last term vanishes if we choose the Lorentz gauge, and the second term is of the form of a
4-divergence. Thus the Lagrangian of this problem differs from the usual one, of Eq. (12.85) by a
4-divergence ∂ α A β ∂ β A α .
The 4-divergence does not change the euations of motion since the fields vanish at the limits of
integration given by the action. Using the generalized Gauss’s theorem or by integrating by parts, we
see the 4-divergence gives zero contribution to the action.
PHY 5347
Homework Set 9 Solutions – Kimel

2. 12.5
a) The system is described by

Background: particle having m, e. Choose ⃗ ⃗ and E


u ⊥ to B ⃗ . We want E
⃗ ′⊥ = 0 = γE + u⃗ × B
⃗ .
c
Thus u⃗c × B⃗ = −E
⃗ ; now B
⃗× ⃗ u×B ⃗ = cE
⃗×B⃗ . Using BAC - CAB on the lhs of the equation gives
⃗ ⃗

u = c E×B2 . Thus from the figure,
B

⃗ ⃗
u = c E ×2 B = c E ẑ

B B
Then, using Eq. 11. 149

⃗ || = 0; ⃗ ′⊥ = 0; ⃗ ′⊥ = 1 B
⃗ ′ = 0; B ⃗ u 2 ⃗
E E B || γ = 1− c B

So

⃗ ′⊥ =
B 1− E 2
⃗ =
B B 2 − E 2 B̂
2
B B2
Now from the class notes,
⃗′ = ⃗
du ′
⃗ B ′ = eB′ , where in this case E ′ is the energy of the particle.
⃗ B ′ , where ω
u′ × ω

dt E
I’ll choose the same boundary conditions as in class, decribed in the figure.
So

u ′⊥ = ω B ′ acosω B ′ t ′ ̂ 3 − sinω B ′ t ′ ̂ 1 
where u ′⊥ t = 0 = ω B ′ a (ie, the BC determine a.

x ′ t ′  = u ′|| t ′ ̂ 2 + a̂ 3 sin ω B ′ t ′ + ̂ 1 cos ω B ′ t ′ 

Consider the inverse Lorentz transformation between the frames,

ct γ ∞γ 0 0 ct ′
z ∞γ γ 0 0 z′
=
x 0 0 1 0 x′
y 0 0 0 1 y′

or
t = γt ′ + ∞γz ′ /c = γt ′ + ∞γa sin ω B ′ t ′ /c ≡ ft ′  → t ′ = f −1 t
So
zt = ∞γcf −1 t + γa sin ω B ′ f −1 t

xt = a cos ω B ′ f −1 t

′ −1
yt = u ||0 f t
⃗ field alone. Then the
b) If |E| ∑ |B|, one can transform to a frame where the field is a static E
solution is as we found in section 12.3 of the text, with the above transformation taking you to the
unprimed frame.
PHY 5347
Homework Set 9 Solutions – Kimel

1. 12.3
⃗ 0 along z, ⃗
a) Take E v 0 along y

Generally

dp ⃗+ ⃗ ⃗ ;
v ×B ⃗
dE = ev⃗ ⋅ E
=e E c
dt dt
⃗ = 0, and E
Since B ⃗ = E 0 ẑ

dp z
= eE 0
dt

dp y
=0
dt
The initial condition ⃗
p0 = mv 0 and the above equations show the subsequent motion is in the
y − z plane. Consistent with this initial condition, we have
p y t = mv 0 ; p z t = eE 0 t

Et = ⃗
p 2 tc 2 + m 2 c 4 = m 2 v 20 c 2 + m 2 c 4 + ceE 0 t 2 = ω 20 + ceE 0 t 2

Using

p = mγv⃗ = E2 ⃗
⃗ v
c

p y t mv 0 c 2
v y t = =
Et/c 2 ω 20 + ceE 0 t 2

mv 0 c 2
∫0
t mv 0 c
yt = dt = sinh −1 t/ρ
ceE 0 ρ +t
2 2 eE 0

ω0
where ρ ≡ ceE 0
. So
mv 0 c tceE
yt = sinh −1  ω 0 0  Eq. (1)
eE 0
Similarly
p z t eE 0 tc 2
v z t = =
Et/c 2 ω 20 + ceE 0 t 2

Thus
2
∫0
t
zt = eE 0 c tdt =c ρ 2 + t 2  − ρ Eq.(2)
ceE 0 ρ2 + t2

b) From Eq. (1)


ω0 eE y eE
t= sinh mv00 c  = ρ sinhky, with k = mv 0c
ceE 0 0

Then from Eq.(2)

z = cρ sinh 2 ky + 1 − 1 Eq.(3)

Let us plot sinh 2 x + 1 − 1

20
18
16
14
12
10
8
6
4
2

0
1 2 x 3 4 5

For small t : t/ρ << 1, and ky << 1. Thus we can taylor expand Eq.(3) and get
z = cρk 2 y 2 /2
which is quadratic in y giving a parabolic shape.

For large t : t/ρ >> 1, and we see the sinh term dominates in Eq.(3) and we get
cρe ky
z
2
which is an exponential shape.
PHY 5347
Homework Set 8 Solutions – Kimel

3. 12.2 (a)

L ′ = L + d λt, ⃗
x
dt

δ ∫ L ′ − Ldt = δλt 2 , ⃗
t2
x  − λt 1 , ⃗
x = 0 → L and L ′ yield the same Euler-Lagrange Eqs. of Mot.
t1

where the last equality follows from the fact that variation at the end points is zero since the end
points are held fixed.

(b) For simplicity of notation in this part, I’m going to set c = 1.


⃗ − eφ with A μ = φ, A
⃗⋅A
L = −m 1 − u 2 + eu ⃗

If A α → A α + ∂ α Λ, then
φ → φ + ∂0Λ

⃗ →⃗
A a−⃗
∇Λ
where the minus sign in the second equation should be noticed. Thus
⃗ − eφ − eu
⃗⋅A
L → −m 1 − u 2 + eu ⃗⋅⃗
∇Λ − e∂ 0 Λ
Now
μ
x = ∂ μ Λt, ⃗
d Λt, ⃗ x ∂x = ∂ Λ + ⃗
∇Λ ⋅ ⃗
u
dt ∂x ∂t ∂t
Or

L → −m 1 − u 2 + eu ⃗ − eφ − e d Λt, ⃗
⃗⋅A x
dt
By the argument of part (a), this Lagrangian gives the same equations of motion as the original
Lagrangian.
PHY 5347
Homework Set 8 Solutions – Kimel

2. 12.1 (a)
q
L = − 1 mu α u α − c u α A α (invariant Lagrangian)
2
Show this Lagrangian gives the correct eqn. of motion, ie, Eq. (12.2)
du α = e F α∞ u ∞
dτ mc
τ2
The Action is A = ∫ Ldτ.
τ1

δA = 0 yields the Lagrange equations of motion

d ∂L − ∂L = 0
dτ ∂u α ∂x α

d ∂L = −m d u α − q ∂A α dx μ
dτ ∂u α dτ c ∂x μ dτ

∂L = − q u ∂ α A ∞
∂x α c ∞
d ∂L ∂L
So dτ ∂u α
− ∂x α
= 0 yields
q q q
m d u α = c u ∞ ∂ α A ∞ − c ∂ μ A α u μ = c u ∞ ∂ α A ∞ − ∂ ∞ A α 

or
d u α = q F α∞ u
mc ∞

PHY 5347
Homework Set 7 Solutions – Kimel

3. 11.15
⃗ || ẑ , so β
From Eq. (11.149), it is clear that we should take β ⃗ =β
⃗⋅E ⃗ = 0. Then
⃗⋅B

⃗+β
E ′ = γE ⃗
⃗×B

⃗ ′ = γB
B ⃗−β ⃗
⃗×E

The vectors in parentheses should make the same angle wrt the x axis θ ′ if they are to be parallel.
This can best be seen from the figure,

From the figure


⃗ ′ = γE 0 î − β2E 0  sin θî + β2E 0 cos θ̂ 
E

⃗ ′ = γcos θ2E 0 î + sin θ2E 0 ̂ − βE 0 ̂ 


B
Thus
2β cos θ 2 sin θ − β
tan θ ′ = =
1 − 2β sin θ 2 cos θ

2 cos θ ⋅ 2β cos θ − 1 − 2β sin θ ⋅ 2 sin θ − β = 0


or
2β 2 sin θ − 5β + 2 sin θ = 0
This quadratic equation has the solution
β= 1 5− 25 − 16 sin 2 θ
4 sin θ
where I’ve chosen the solution which give β = 0 if θ = 0.

If θ << 1, then β → 0, and the original fields are parallel.

If θ → π/2 then β = 1
4 5 − 3 = 1/2. γ = 2
3

⃗ ′ = 0 + Oθ − π ̂
E
2

⃗′ = B
B ⃗ ′ = γ2E 0 ̂ − 1 E 0 ̂  = γE 0 3 ̂
2 2
So in these two limits, the fields are parallel to the x and y axes, respectively.
PHY 5347
Homework Set 7 Solutions – Kimel

2. 11.5

From the class notes, and Eq. (11.31), we know


u ′|| + v u ′⊥
u || = vu ′||
; u⊥ = vu ′||
1+ c2
γ1 + c2

and
vu ′||
dt = dt ′ γ1 + 
c2
Thus taking the differential of the first equation above and using the second equation for dt,
vu ′|| 3/2
du || 1 + a ′|| − u ′|| + v cv2 a ′|| 1− v2
≡ a || = c2
vu ′||
= c2
vu ′|| 3
a ′||
dt
γ1 + c2
3 1 + c2

Similarly,
vu ′||
1+ a ′⊥ − u ′⊥ v
a ′||
du ⊥ ≡ a = c2 c2

dt vu ′||
γ 2 1 + c2
3

This is equal to the expression we want to prove,


v2
1−
du ⊥ ≡ a =

c2
vu ′||
a ′⊥ + ⃗
v × a
⃗′ × ⃗
u′ 
dt
1 + 3 c2
c2

since the BAC - CAB theorem shows that


vu ′||
a ′⊥ + ⃗
v × a
⃗′ × ⃗
u′  = 1+ a ′⊥ − u ′⊥ v2 a ′||
c2 c2 c
PHY 5347
Homework Set 7 Solutions – Kimel

1. 11.3 Let us just focus on the 0, 1 component transformation, since the 2, 3 components
remain unchanged, if we take the relative velocities between Lorentz frames to be along the x -
direction. We want to relate a single Lorentz transformation to two sequential transformations as
described by

Thus we require
A = A2A1
where A is a Lorentz transformation. Rewritten explicitly, the above equation reads

γ −∞γ γ2 −∞ 2 γ 2 γ1 −∞ 1 γ 1
=
−∞γ γ −∞ 2 γ 2 γ2 −∞ 1 γ 1 γ1

γ2γ1 + ∞2γ2∞1γ1 −γ 2 ∞ 1 γ 1 − ∞ 2 γ 2 γ 1
=
−γ 2 ∞ 1 γ 1 − ∞ 2 γ 2 γ 1 γ2γ1 + ∞2γ2∞1γ1

So
γ = γ2γ1 + ∞2γ2∞1γ1

∞γ = γ 2 ∞ 1 γ 1 + ∞ 2 γ 2 γ 1

∞γ γ ∞ γ + ∞2γ2γ1 ∞1 + ∞2
∞= γ = 2 1 1 =
γ2γ1 + ∞2γ2∞1γ1 1 + ∞2∞1
Or

v = v 1 +v 1vv22
1 + c2

as required.
PHY 5347
Homework Set 6 Solutions – Kimel

1. 10.1
a) Let us first simplify the expression we want to get for the cross section. Using n̂ 0 = ẑ ,
dσ ̂ , n̂ , n̂  = k 4 a 6 5 − |̂ 0 ⋅ n̂ |2 − 1 |n̂ ⋅ ẑ × ̂ |2 − ẑ ⋅ n̂
dΩ 0 0 4 4 0

Orienting the system as

and using
̂ 0 = α 0 x̂ + ∞ 0 , with |α 0 |2 + |∞ 0 |2 = 1

n̂ = cos θẑ + sin θx̂


then
dσ ̂ , n̂ , n̂  = k 4 a 6 5 − |α |2 sin 2 θ − 1 |∞ 0 |2 sin 2 θ − cos θ
dΩ 0 0 4 0
4
Using the result for the perfectly conducting sphere Eq. (10.14)
2
dσ n̂ , ̂, ̂ 0 , n̂ 0  = k 4 a 6 ̂ ∗ ⋅ ̂ 0 − 1 ẑ × ̂  ⋅ n̂ × ̂ ∗ 
0
dΩ 2
Using ̂ ⊥ =
2 2
dσ n̂ , ̂ ⊥ , ̂ 0 , n̂ 0  = k 4 a 6 ∞ 0 − 1 ∞ 0 cos θ = k 4 a 6 |∞ 0 |2 1 − 1 cos θ
dΩ 2 2
Similarly ̂ || = x̂ ′
2 2
dσ n̂ , ̂ , ̂ , n̂  = k 4 a 6 α cos θ − 1 α = k 4 a 6 |α 0 |2 cos θ − 1
|| 0 0 0
dΩ 2 0 2
By definition
dσ ̂ , n̂ , n̂  = dσ n̂ , ̂ ⊥ , ̂ 0 , n̂ 0  + dσ n̂ , ̂ || , ̂ 0 , n̂ 0 
dΩ 0 0 dΩ dΩ

2 2
= k 4 a 6 |∞ 0 |2 1 − 1 cos θ + |α 0 |2 cos θ − 1
2 2
which simplifies to
dσ ̂ , n̂ , n̂  = k 4 a 6 5 − |α |2 sin 2 θ − 1 |∞ |2 sin 2 θ − cos θ
dΩ 0 0 4 0
4 0
using |α 0 |2 + |∞ 0 |2 = 1, and cos 2 θ = 1 − sin 2 θ.

b) If ̂ 0 is linearly polarized making an angle φ with respect to the x axis, then


̂ 0 = α 0 x̂ + ∞ 0 = cos φx̂ + sin φ , so α 0 = cos φ, ∞ 0 = sin φ
Then from part a)
dσ ̂ , n̂ , n̂  = k 4 a 6 5 − |α 0 |2 sin 2 θ − 1 |∞ 0 |2 sin 2 θ − cos θ
dΩ 0 0 4 4

= k 4 a 6 5 − cos 2 φ sin 2 θ − 1 sin 2 φ sin 2 θ − cos θ


4 4
Using cos2φ = cos 2 φ − sin 2 φ, this expression simplifies to
dσ ̂ , n̂ , n̂  = k 4 a 6 5 1 + cos 2 θ − 3 sin 2 cos 2φ − cos θ
dΩ 0 0 8 8
as desired.
PHY 5347
Homework Set 5 Solutions – Kimel

2. The system is described by

It = I 0 cos ωt = ReI 0 e −iωt 


Jt = 1a Itδr − aδcos θφ̂

where I determined the normalization constant 1


a by the condition ∫ ⃗ ⃗=I
J ⋅ da


Jt = Re Ia0 δr − aδcos θφ̂ e −iωt →⃗
J = Ia0 δr − aδcos θφ̂

⃗ and E
We use the general expression for H ⃗ in the radiation zone given by Eq.(9.149). Since this
system has no net charge density and there is no intrisic magnetization, the the expansion coefficients in
these equations are given by

a E l, m = k2 ∫ Y m∗
l ik
⃗r ⋅ ⃗
J j l krd 3 x
i ll + 1

a M l, m = k2 ∫ Y m∗ ⃗
l ∇⋅
⃗r × ⃗
J j l krd 3 x
i ll + 1

a) ⃗r ⋅ ⃗
J = 0 in the first equation, so there is no electric multipole radiation. In spherical coordinates
⃗r × ⃗
J = −aJθ̂

Using the formulas for ⃗ ⃗ in spherical coordinates given in the back of the book,
∇⋅A
⃗ J = − 1 ∂ J sin θJ = − cos θ J − ∂ J
∇ ⋅ ⃗r × ⃗
sin θ ∂θ sin θ ∂θ
The first term does not contribute, because cos θ = 0, while the second term can be written, using
the chain rule,

∇ ⋅ ⃗r × ⃗
J = sin θ ∂ J
∂ cos θ
The problem has azimuthal symmetry, so m = 0. Realizing derivatives of δ −functions are defined
by integration by parts,
δ m0 k 2
a M l, m = ∫ Y 0∗l sin θ ∂ J j krd 3 x =
∂ cos θ l
ik 2 ∫ ∂ sin θY 0∗  Jj l krd 3 x
∂ cos θ l
i ll + 1 ll + 1

a M l, 0 = i2πk 2 I 0 a 2 j ka ∂ sin θY 0 |


a l l cos θ=0
ll + 1 ∂ cos θ

a M l, 0 = i2πk 2 I 0 a 2 j l ka1 − x 2  1/2 d Y 0 x|


a x=0
ll + 1 dx l

Since Y 0l x is either an even or odd polynomial in x, then only odd l contribute to a M l, 0. This
determines the expansion coefficients, and thus H ⃗ and E⃗ in the radiation zone are known through
Eq.(9.149). The power distribution is given by Eq. (9.151)

b) From our previous answers, we see a E l, m = 0, and that the lowest magnetic multipole
contribution is a M 1, 0.
2
a M 1, 0 = i2πk I 0 aj 1 ka1 − x 2  d Y 01 x|x=0
1/2

2 dx

Using

j 1 ka → ka ; d Y 0 x| = 3
3 dx 1 x=0 4π

a M 1, 0 = i2πk 3 I 0 a 2 1 = ik 3 2 M
l0
24π 3

1
i2πk 3 I 0 a 2
M l0 =
24π
= 3 I πa 2
ik 3
2 4π 0
3

Note that you would get the same answer, if you used Eq. (9.172) directly.
From Eq. (9.151)
2
dP = Z 0 2πk Fa 3 2 1 3 sin 2 θ = 1 Z 0 k 4 I πa 2  2 sin 2 θ
0
dΩ 2k 2 24π 8π 32π 2
If we compare this result with the one that we get for an elementary magnetic dipole, which is
given by Eq. (9.23)
with the substitution ⃗
p→m ⃗ /c,
dP = 1 Z 0 k 4 |m
⃗ |2 sin 2 θ
dΩ 32π 2
Thus we may identify
⃗ | = I 0 πa 2
|m
as would be expected.
PHY 5347
Homework Set 5 Solutions – Kimel

1. The system is described by

and is azimuthally symmetric


Rθ = R 0 1 + ∞tP 2 cos θ; ∞t = ∞ 0 cos ωt; kR << 1

∫ ρr 2 drdφd cos θ = 2π ∫ −1 d cos θρ ∫ 0


1 Rθ
Q= r 2 dr

= 2π ρ ∫ R 30 1 + 3∞P 1 P 2 d cos θ + O∞ 2  = 4π ρR 30 → ρ = 3 3 Q


1

3 −1 3 4πR 0
where I’ve used the fact that 1 = P 0 . Since the system is azimuthally symmetric, Q lm = δ m0 Q l0 .

Q lm = 2πρδ m0 ∫ dxY 0l ∫ ∫ −1 dxR l+3


1 Rθ 2πρδ m0 1
r l+2 dr = 0 1 + l + 3∞P 2 Y l
0
−1 0 l+3

Using Y 0l = 2l+1

P l and 1 = P 0 ,

Q lm =
2πρδ m0 2l + 1 R l+3 2δ + l + 3∞ 2 δ
l+3 4π 0 l0
2l + 1 l2
Notice that the l = 0 term is time independent and thus does not contribute to the radiation.
Next consider the l = 2 term.

Q 20 t = 2 π ρ 5 R 50 ∞ = ρ = 3 3 Q 2 π ρ 5 R 50 ∞ = 3 R 2 Q∞t
0
5 4πR 0 5 20π

Q 20 t = Re 3 R 2 Q∞ 0 e −iωt
0
20π
Q 20 = 3 R 2 Q∞
0 0
20π

dP2, 0 Z ⃗ 20
= 02 |a2, 0|2 X
2
dΩ 2k

a E 2, 0 = ck 4 3 Q
i5 × 3 2 20

⃗ 20
X
2
= 15 sin 2 θ cos 2 θ

2
dP2, 0 ck 4
= Z 02 3 Q × 15 sin 2 θ cos 2 θ
dΩ 2k 5 × 3 2 20 8π

2
2
4 2 ck 4  3
R 20 Q∞ 0
1 Z 0 |ck Q 20 | 1 Z 20π
= π sin θ cos θ =
2 2 0
π sin 2 θ cos 2 θ
160 k 2 160 k 2

= 9 Z 0 k 6 c 2 R 40 Q 2 ∞ 20 sin 2 θ cos 2 θ
3200π 2

Z k 6 c 2 R 40 Q 2 ∞ 20 × 2π ∫ 1 − x 2 x 2 dx =
1
P= 9 9 Z 0 k 6 c 2 R 40 Q 2 ∞ 20 × 2π × 4
2 0
3200π −1 3200π 2 15

P= 3 Z k6c2R4Q2∞2
2000π 0 0 0
PHY 5347
Homework Set 4 Solutions – Kimel

1. 9.11 We are working with small sources in the radiation zone.

From the notes and Eqs (9.170) and (9.172),

Q lm = ∫ r l Y m∗
l ρd x
3

M lm = − 1
l+1
∫ r l Y m∗ ⃗
l ∇⋅
⃗r × ⃗
J d3x

a) Electric Dipole Radiation:

Q 1m = ∫ rY m∗
1 ρd x
3

= ∫ rδxδy−qδz − a cos ω 0 t − qδz + a cos ω 0 t + 2qδzY m∗


1 dxdydz

1 0, φ + Y 1 π, φ = −qaδ m0 cos ω 0 tY 1 0 + Y 1 π = 0


= −qa cos ω 0 tY m∗ m∗ 0 0

b) Magnetic Dipole Radiation: Since the particles move in an orbit with no area,
⃗r × ⃗
J = 0 → M 1m = 0
c) Electric Quadrupole Radiation:

Q 2m = ∫ r 2 Y m∗
2 ρd x
3

= −qδ m0 a 2 cos 2 ω 0 tY 02 θ = 0 + a 2 cos 2 ω 0 tY 02 θ = π

= −qδ m0 a 2 Y 02 0cos 2ω 0 t + 1
where I have used cos 2 ω 0 t = cos 2ω 0 t + 1/2. Thus the Fourier Series decomposition of this
moment yields terms with frequency 0, and 2ω 0 . The first term does not contribute to radiation, and
the second can be written
Q 20 t = Re−2qa 2 Y 02 0e −2iω 0 t 

so Q 20 = −2qa 2 Y 02 0 is the quantity that is used in the radiation formulas of Jackson. Using Eq.
(9.151)
⃗ 20
dP 2, 0 = Z 0 |a2, 0|2 X 2
dΩ 2k 2
and from Eq. (9.169)

a2, 0 = ck 4 3 Q 20 = ck 4 3 −2qa 2  5
i5 × 3 2 i5 × 3 2 4π

where I have used Y 02 0 = 5



. Thus

|a2, 0|2 = 1 c2k8q2a4


30π

dP 2, 0 = Z 0 1 c2k8q2a4 15 sin 2 θ cos 2 θ = 1 Z k 6 c 2 q 2 a 4 sin 2 θ cos 2 θ


0
dΩ 2k 2 30π 8π 32π 2

P2, 0 = 2π Z k 6 c 2 q 2 a 4 ∫ 1 1 − x 2 x 2 dx = 2π Z k 6 c 2 q 2 a 4 × 4
0 0
32π 2 −1 32π 2 15

P2, 0 = 1 Z k6c2q2a4
60π 0
PHY 5347
Homework Set 3 Solutions – Kimel

3. 9.3

Since the problem has azimuthal symmetry, we can expand Vr⃗, t (in the radiation zone) in terms
of Legendre polynomials:

Vr⃗, t = > b l tr −l−1 Pl cos θ


l

Using the orthogonality of the Legendre polynomials, the leading term of the expansion in the
radiation zone will be the l = 1 term.

b 1 t = 3 R 2 ∫ xVr⃗, tdx = 3 VR 2 cos ωt


1

2 −1 2
So,

Vr⃗, t = 3 VR 2 cos ωt /r 2 = ⃗
p ⋅ r̂
cos ωt = Re

p ⋅ r̂ −iωt
e
2
2 r r2
with ⃗
p= 3
2
VR 2 ẑ , which should be used in the radiation formulas developed in lecture.

dP = c 2 Z 0 k 4 |p
⃗ |2 sin 2 θ
dΩ 32π 2

2 4
P = c Z 0 k2 8π = 1 c 2 Z 0 k 4 |p
⃗ |2
32π 3 12π
with ⃗
p= 3
2
VR 2 ẑ .
PHY 5347
Homework Set 3 Solutions – Kimel

3. 9.2 First consider a rotating charge which is at an angle α at time t = 0.

Compared to the lecture notes for this problem, where we assumed α = 0, we should let
ωt → ωt + α. Thus using the result developed in class, we can write for this problem

1 i 0
Q α t = Re 3 qd 2 i −1 0 e −i2a e −i2ωt
2
0 0 0

From the figure


Q tot t = Q α1 t + Q α2 t + Q α3 t + Q α4 t

1 i 0
= Re 3 qd 2 −e −i π2 + e −i 3π2 − e −i 5π2 + e −i 7π2 i −1 0 e −i2ωt
2
0 0 0

1 i 0
Q tot = 3 qd 2 4i i −1 0
2
0 0 0

Thus from the class notes


dP = c 2 Z 0 k 6 qd 2 3 2
161 − cos 4 θ = 1 c 2 Z k 6 q 2 d 4 1 − cos 4 θ
0
dΩ 1152π 2 2 32π 2

c2Z0k6 2
P= qd 2 3 16 = 1 c2Z0k6q2d4
360π 2 10π
And, of course, the frequency of the radiation is 2ω.
PHY 5347
Homework Set 3 Solutions – Kimel

3. 9.1 a)

ρx⃗, t = qδzδy − sin ω 0 tδx − d cos ωt


To illustrate the equivalence of the two methods, I’ll consider the lowest two moments.

n = 0 : Qt = ∫ ρx⃗, td 3 x = q = Reqe −i0⋅ωt 

n=1:⃗
pt = ∫ ρx⃗, tx⃗d 3 x = qdî cos ωt + sin ωt = Reqdî + i e −i1⋅ωt 

So we identify ⃗
p = qdî + i  as the quantity to be used in Jackson’s formulas.
Arbitrary n: The n’th multipoles will contribute with maximum frequencies of ω n = nω.

b) The proof that we can write



ρx⃗, t = ρ 0 x⃗ + > Re2ρ n x⃗e −inωt 
n=1

with
τ
ρ n x⃗ = 1τ ∫ 0 ρx⃗, te inωt dt
was presented in lecture and will not be repeated here.
c) We have already calculated the n = 0, 1 moments by the method of part a). Now we
compute these moments by the method of part b).
n=0:
τ
ρ 0 x⃗ = 1τ ∫ 0 qδzδy − sin ω 0 tδx − d cos ωtdt
τ
Q= ∫ ρ 0 x⃗d 3 x = q
τ ∫ 0 dt ∫ d 3 xδzδy − sin ω 0 tδx − d cos ωt = q
n=1:
τ
ρ 1 x⃗ = 1τ ∫ 0 qδzδy − sin ω 0 tδx − d cos ωte iωt dt
τ

px⃗ = ∫ d 3 xx⃗2ρ 1 x⃗ = 2q
τ ∫ 0 dt ∫ d 3 xx⃗δzδy − sin ω 0 tδx − d cos ωt
τ
=
2qd
τ ∫ 0 dte iωt î cos ωt + sin ωt = qdî + i 

as before.
PHY 5347
Homework Set 2 Solutions – Kimel

1. 8.5 a)

For the TM modes, we saw in class the resonance frequencies are


TM:

p = 0, 1, 2, . . . .
1 x 2mn p2π2
ω mnp = + m = 0, 1, 2, . . . .
μ R2 L2
n = 1, 2, 3, . . . .

TE:

p = 1, 2, 3, . . . .
1 x ′2 p2π2
ω ′mnp = mn
+ m = 0, 1, 2, . . . .
μ R2 L2
n = 1, 2, 3, . . . .

Thus

ω mnp x 2mn p2π2


1
= 2
+
μ
R L2

ω ′mnp x ′2 p2π2
1
= mn
+
μ
R2 L2

The lowest four frequencies are (in these units)

ω 010 = 2. 405
ω 110 = 3. 832
2
ω ′111 = 1. 841 2 + π 2 R
L
2
ω ′211 = 3. 054 2 + π 2 R
L

2. 405, 3. 832, 1. 841 2 + π 2 x 2 , 3. 054 2 + π 2 x 2


10

0
0.2 0.4 0.6 0.8 1
x 1.2 1.4 1.6 1.8 2

where x = R/L.

The answer is ”No.” ω ′111 and ω 010 cross when

1. 841 2 + π 2 x 2 = 2. 405
or x = 0. 492 58. For frequencies smaller than this cross over frequency, ω ′111 is lowest, whereas for
larger frequencies, ω 010 is lowest.
PHY 5347
Homework Set 1 Solutions – Kimel

1. 8.4

a) TM:

∇ 2t + γ 2 ψ = 0; ψ|B = 0; E z = ψx, ye ±ikz−iωt ; B z = 0


Since we have a node along y = x, then we just take the antisymmetrized version for the square
waveguide, developed in class, ie,
nπy mπy
ψx, y = E 0 sin mπx nπx
a  sin a  − sin a  sin a 
Again
2
γ 2mn = cπ2 m 2 + n 2 , m, n = 1, 2, 3. . . . , but m ≠ n
a
TE:
∂ψ
∇ 2t + γ 2 ψ = 0; | = 0; H z = ψx, ye ±ikz−iωt ; E z = 0
∂n B
∂ψ
Now the BC require |
∂n B
= 0, but using a 45 0 rotation of coordinates, we see

∂ = 1 − ∂ + ∂
∂n 2 ∂x ∂y

Thus the combination


nπy mπy
ψx, y = H 0 cos mπx nπx
a  cos a  + cos a  cos a 
satisfies the above BC on the diagonal, as you can see by direct substitution.
2
γ 2mn = cπ2 m 2 + n 2 , m, n = 0, 1, 2, 3. . . . , but m ≠ n = 0
a
b) The lowest cutoff freq. are: TM: ω 12 or ω 21 . TE: ω 01 or ω 10 . From Eq. (8.63) in the text
1/2
∞ 12 TM  ω
1 − ω 212 /ω 2

1/2
ω ω 201
∞ 01 TE  1+
1 − ω 212 /ω 2 ω2
For the square wave guide, we don’t have the antisymmetrization, but the formulas for the cutoff
frequencies are the same without the present restrictions on m and n. So for the square guide, the cut
off frequencies are
TM: ω 11
TE: ω 01 (as before)
PHY 5347
Homework Set 1 Solutions – Kimel

1. 8.3

z
y

a)
∇ 2t + γ 2 ψ = 0, ψ = E z TM or ψ = H z TE
As in class, we will use cylindrical coordinates, and assume
ψρ, φ = RρQφ
We get the two equations
∂ 2 Qφ = −m 2 Qφ with solns Qφ = e ±imφ , m = 0, 1, 2, . . .
∂φ 2

d 2 Rx + 1 dRx + 1 − m 2 Rx Bessel eqn.


dx 2 x dx x2
with regular solutions J m x, and singular solution (which we reject as nonphysical) N m x. Here
x = γρ.
Solutions:
TM: BC: J m x mn  = 0, and
E z ρ, φ = E 0 J m γ mn ρe ±imφ , m = 0, 1, 2, . . . . ; n = 1, 2, 3, . . . ; γ mn = x mn /R
Lowest cutoff frequencies:
γ mn
ω mn = = x mn
μ R μ

Using the results of Jackson, p. 114,


x 0n = 2. 405, 5. 52, 8. 654, . . .
x 1n = 3. 832, 7. 016, 10. 173, . . . .
x 2n = 5. 136, 8. 417, 11. 620, . . . .

TE: BC: J ′m x ′mn  = 0, and


E z ρ, φ = E 0 J m γ ′mn ρe ±imφ , m = 0, 1, 2, . . . . ; n = 1, 2, 3, . . . ; γ ′mn = x ′mn /R
Lowest cutoff frequencies:
γ ′mn x ′mn
ω mn = =
μ R μ

Using the results of Jackson, p. 370,

x ′0n = 3. 832, 7. 016, 10. 173, . . .


x ′1n = 1. 841, 5. 331, 8. 536, . . . .
x ′2n = 3. 054, 6. 706, 9. 970, . . . .

From the above we see the lowest cutoff frequency is the TE mode
ω ′11 = 1. 841K, with K = 1/ R μ

The next four lowest cutoff frequencies are:

ω 01 = 2. 405K = 1. 31ω ′11



ω 21 = 3. 054K = 1. 66ω ′11
ω ′01 = 3. 832K = 2. 08ω ′11
ω 11 = 3. 832K = 2. 08ω ′11

b) From Eq. (8.63) in the text


1/2

ω ξ λ + η λ ωωλ
2
∞λ  ω2
1 − ω λ2

For TM modes, η λ = 0, and for TE mode, ξ λ + η λ is of order unity. So for comparison purposes,
I’ll take
1/2
2
∞ 11 TE = f 1 x = x 1 + 1. 841
1.841 2 2
1− x2
x

1/2

∞ 01 TM = f 2 x = x
2.405 2
1− x2

where I’ve expressed the functions in terms of x = ω/K.


1/2
1.841 2
1+ x2
x
2
1− 1.841
2
x

14

12

10

0 2
4 x6 8 10

1/2
x
2
1− 2.405
2
x

14

12

10

0
3 4 5 6x 7 8 9 10
PHY 5346
Homework Set 13 Solutions – Kimel

2. 7.4 We have a nonpermeable conducting material, so μ = μ 0 , and we have J = σE, where σ is


the conductivity. The following figure describes the system:

The two boundary conditions that we must satisfy for plane waves are
E 0 + E ′′0 − E ′0 = 0

kE 0 − E ′′0  − k ′ E ′0 = 0
Or
E ′′0 ′
= k − k′
E0 k+k
We must take into account the fact that ⃗ ⃗ . Adding in this term in Maxwell’s equations for a
J = σE
plane wave, we get
k= ω
c

σ
k ′2 = μω 2 1 + i ω

Thus we can write


k′ = μ ωα + iβ

with
1/2
σ 2
1 +  ω  +1
α=
2
1/2
σ 2
1 +  ω  −1
β=
2

Thus
E ′′0 1 − μ 0 cα − i μ 0 cβ
=
E0 1 + μ 0 cα + i μ 0 cβ

1) For a very poor conductor σ is very small, so keeping only first order in σ
1/2
σ 2
1 +  ω  +1
α= ≈1
2

1/2
σ 2
1 +  ω  −1 σ
β= ≈
2 2ω

σ
2) For the case of a very good conductor, ω >> 1, so
2

α≈ σ = μ 0 ωδ 2
= 1
2ω 2ω ωδ μ 0 

β≈ σ = 1
2ω ωδ μ 0 

where I have used (5.165) to relate the conductivity to the skin depth.

σ= 2
μ 0 ωδ 2

E ′′0 1− c
− i ωδ
c
δ− c
− i ωc
= ωδ
= ω
≈ −1 + ω 2 ω
c 1 + i δ = −1 + c 1 − iδ
E0 1+ c
ωδ
+ i ωδ
c
δ+ c
ω + i ωc

E ′′0 2
ωδ 2
R= = −1 + δω/c 2 + c ≈ 1 − 2δω/c
E0
PHY 5346
Homework Set 13 Solutions – Kimel

1. 7.2
a) The figure describes the muliple internal reflections which interfere to give the overall
reflection and refraction:

For the ij interface I shall use the notation


E ′0
r ij = = n 2n i
E0 i + nj

E ′′0 n −n
R ij = = ni + nj
E0 i j

Thus from the figure


E ′′0 = E 0 R 12 + r 12 E 0 R 23 r 21 e iφ + r 12 E 0 R 23 R 21 R 23 r 21 e i2φ +. . . .


E ′′0 = E 0 R 12 + r 12 E 0 R 23 r 21 e iφ >R 21 R 23 e iφ 
n

n=0

E ′′0 = E 0 R 12 + r 12 r 21 R 23
−iφ
e − R 21 R 23 
Similarly
E ′0 = E 0 r 12 r 23 + E 0 r 12 R 23 R 21 r 23 e iφ +. . . .
r 12 r 23
E ′0 = E 0
1 − R 21 R 23 e iφ
where the phase shift for the internally reflected wave is given by
2π2d ωn 2 2d
φ= =
λ2 c

Now for a plane wave

S i = 1 |E 0i |2
2v i
Thus
|E ′′ |
2
′′
R= S = 02
S |E 0 |

′ ′
T = vv 13 S = nn 3 S
S 1 S
From the above

2r 12 r 21 R 23 R 12 cos φ − R 21 R 23  + R 12 r 21 R 23  2
R= R 212 +
1 + R 21 R 23  2 − 2R 21 R 23 cos φ

r 12 r 23  2
T = nn 3
1 1 + R 21 R 23  2 − 2R 21 R 23 cos φ

Since these two equations are simple functions of φ, which is linearly proportional to the
frequency, they are simple functions of frequency which you should plot.

b) Since in part a) we used the convention that the incident wave is from the left, I will rephrase
this question so that n 1 is are, n 2 is the coating, and n 3 is glass. In this case, we will have
n 1 < n 2 < n 3 , and R 21 R 23 < 0. Thus for T to be a maximum, from the above equation cosφ = −1, or
φ = π.
2π2d λ
φ= =π→d= 2
λ2 4
where λ 2 is the wavelength in the medium = λn 21
PHY 5346
Homework Set 12 Solutions – Kimel

2. 7.1 I shall apply Eqs.(26), (27), and (28)


s0 + s1 = a s0 − s1
= a2 δ l = δ 2 − δ 1 = sin −1 s3
1
2 2 2a 1 a 2

s0 + s3 = a s0 − s3
= a− δ c = δ − − δ + = sin −1 s2
+
2 2 2a + a −
a) s 0 = 3, s 1 = −1, s 2 = 2, s 3 = −2

a 1 = 1, a 2 = 2

δ l = sin −1 −2 = − 1 π rad
2 2 4

a+ = 1 , a− = 5
2 2

δ c = sin −1 2 = 1. 107 1 rad


1 5
2 2
2

b) s 0 = 25, s 1 = 0, s 2 = 24, s 3 = 7

a1 = 25 , a = 25
2
2 2

δ l = sin −1 s3 = sin −1 7 = 0. 283 79 rad


2a 1 a 2 2 25 25
2 2

a+ = 32 = 4, a− = s0 − s3 = 3
2 2

δ c = δ − − δ + = sin −1 24 = 1 π rad
24 × 3 2
To plot the two cases Re E x ≡ X = cos x, ReE y ≡ Y = rcoxx − δ l , where r = a 2 /a 1 and x = ωt.
Case a) cos x, 2 cosx + π4 
1

0.5

-1 -0.8 -0.6 -0.4 -0.2 0.2 0.4 0.6 0.8 1


-0.5

-1

Case b) cos x, cosx − 0. 283 79

1
0.8
0.6
0.4
0.2

0
-1 -0.8 -0.6 -0.4 -0.2 0.2 0.4 0.6 0.8 1
-0.2
-0.4
-0.6
-0.8
-1
PHY 5346
Homework Set 11 Solutions – Kimel

3. 5.29 The system is described by

This problem is very much like 5.26, except the wires are superconducting. We know from section
5.13 that the magnetic field within a superconductor is zero. We will be using

W= 1
2
∫ ⃗J ⋅ A⃗d 3 x = l ∫J Ada + J Ada 
2 a a b b

Using the same arguments as applied in problem 5.26,


μI ρ μI ρ2
− 2π ln +C = − 4π ln + 0 on the outside
Az = R R2

0, on the inside

Thus if we consider the second term l


2
∫ J b Ada b ,
l
2
∫ J b Ada b = l I
2 πb 2
∫A out ρ a  + A in ρ b ρ b dρ b dφ

μI μ
2π ∫ ln d 2 ρ b dρ b = l
b 2
≃ l I2 2 ln da I 2
2 πb 4π 0 a 2 4π
The first term l
2
∫ J a Ada a is equal to
μ
l
2
∫ J a Ada a = l
2 4π
2 ln d I 2
b
Thus
μ 2
W= l 2 ln d I 2 = l L I 2
2 4π ab 2 l
so
L = μ 2
2 ln d
l 4π ab
Now using the methods of problem 1.6, assuming the left wire has charge Q, and the right wire charge
−Q, we find
Q Q
∫b ∫b
d−a d−a 2
φ 12 = Edr = l 1 + 1 dr ≃ l ln d
2π r d−r 2π ab
Q
C = l = 2π
l φ 12 2
ln dab

Thus
L × C = μ 2
2 ln d × 2π 2 = μ
l l 4π ab ln dab
PHY 5346
Homework Set 11 Solutions – Kimel

2. 5.27 The system is described by

Using Ampere’s law in integral form

∫ B⃗ ⋅ dl⃗ = μ 0 I enclosed
we get
μ0I ρ
B= ,ρ<b
2π b 2

μ0I 1
B= ,b<ρ<a
2π ρ

B = 0, ρ ∑ a
Now the energy in the magnetic field is given by ( l is the length of the wires)

W= 1
2
∫ B⃗ ⋅ H⃗ d 3 x = 1
2μ 0
∫ B2d3x

μ0I 2
ρ 2
l 2π ∫ ρdρ + 2π ∫
b a 2
= 1 1 ρdρ
2μ 0 2π 0 b2 b
ρ

1 μ0I 2
= lπ 1 + 2 ln a = l L I2
2μ 0 2π 2 b 2 l

μ
→ L = 0 1 + 2 ln a
l 4π 2 b
If the inner wire is hollow, B = 0, ρ < b, so
L = μ 0 ln a
l 2π b
PHY 5346
Homework Set 11 Solutions – Kimel

1. 5.26 The system is described by

Since the wires are nonpermeable, μ = μ 0 . The system is made of parts with cylindrical
symmetry, so we can determine B using Ampere’s law.
⃗ ⃗ = μ 0⃗
∇×B J, or ∫ B⃗ ⋅ dl⃗ = μ 0 ∫ ⃗J ⋅ da⃗
On the outside of each wire,
μ0I
∫ B⃗ ⋅ dl⃗ = B2πρ = μ 0 I → B out = 2πρ
On the inside of each wire

∫ B⃗ ⋅ dl⃗ = B2πρ = μ 0 I ρR 2 , μ0I ρ


2
B in = with R = a, b
2π R 2
From the right-hand rule, the B from each wire is in the φ̂ direction. From the above figure, using
⃗ is in the ±ẑ direction. Since ⃗
the general expression for the vector potential, we see A ⃗ =B
∇×A ⃗,

B z = − ∂ A z → A z = − ∫ B z dρ
∂ρ
Thus
μ0I ρ μ0I ρ2
− 2π
ln R
+C =− 4π
ln R2
+ 1 on the outside
Az =
− μ4π0 I ρ2
R2
, on the inside

where I’ve determined C = 1/2, from the requirement that A z be continuous at ρ = R. Let l be the
length of the wire. Then we know the total potential energy is given by

W= 1
2
∫ ⃗J ⋅ A⃗d 3 x = l ∫J Ada + J Ada 
2 a a b b

Consider the second term l


2
∫ J b Ada b . The system is pictured as

From the figure

⃗a = ⃗
ρ ⃗ b,
d+ρ ρ 2a = d 2 + ρ 2b − 2dρ b cos φ
so, since J b = I
πb 2

l
2
∫ J b Ada b = l I
2 πb 2
∫A out ρ a  + A in ρ b ρ b dρ b dφ

μ I ρ 2a ρ 2b
= l I2 0
2 πb 4π
∫ ln
a2
+ 1 −
b2
ρ b dρ b dφ

μ I ρ2
≃ l I 2 0 2π ∫ ln d 2 + 1 − 2b
b 2
ρ b dρ b
2 πb 4π 0 a b

μ I 2 μ0
= l I 2 0 2π 1 b 2 1 + 2 ln d 2 = l 1 + 2 ln d I 2
a
2 πb 4π 4 a 2 4π 2
The first term l
2
∫ J a Ada a is equal to
μ0
l
2
∫ J a Ada a = l
2 4π
1 + 2 ln d I 2
2 b
Thus
μ0 2
W= l 1 + 2 ln d I 2 = l L I 2
2 4π ab 2 l
or
L = μ 0 1 + 2 ln d 2
l 4π ab
PHY 5346
Homework Set 9 Solutions – Kimel

3. 5.19 The system is described by

The effective volume magnetic charge density is zero, since M ⃗ is constant within the cylinder. The
⃗ from Eq. (5.99)) is M 0 , on the top surface and −M 0 on the
effective surface charge density (n̂ ⋅ M
bottom surface. From the bottom surface the potential is (for z ∑ 0
ρdρ
≠ b = 1 −M 0 2π ∫
a
1/2
= − M0 a 2 + z 2  − z
4π 0 ρ + z 
2 2 2

By symmetry, the potential from the top surface is (on the inside)

≠t = M0 a 2 + L − z 2 − L − z
2
The total magnetic potential is
M0 M
≠ = ≠b + ≠t = − a 2 + z 2  − z + 0 a 2 + L − z 2 − L − z
2 2
So, on the inside of the cylinder,

Hz = − ∂ − 0
M M0
a 2 + z 2  − z + a 2 + L − z 2 − L − z
∂z 2 2

Hz = −
M0
2− z − L−z
2 a + z 2 
2
a + L − z 2
2

while above the cylinder,


Hz = − M0 − z + z−L
2 a + z 2 
2
a + L − z 2
2

with a similar expression below the cylinder.


⃗ = μ0 H
B ⃗ +M

Thus inside the cylinder,

Bz = μ0 −
M0
2− z − L−z + M0
2 a + z 2 
2
a + L − z 2
2

μ0M0 z L−z
Bz = +
2 a + z 2 
2
a + L − z 2
2

while above the cylinder,

μ0M0 z z−L
Bz = −
2 a + z 2 
2
a + L − z 2
2

First we plot B z in units of a for L = 5a

1
2
z
+ 5−z
if z < 5
1+z 2 1+5−z 2
gz =
1
2
z
− z−5
if 5 < z
1+z 2 1+5−z 2

gz

0.8

0.6

0.4

0.2

0
2 4 z 6 8 10

And similarly, H z in units of a for L = 5a.


− 12 2− z
− 5−z
if z < 5
1+z 2 1+5−z 2
fz =
− 12 − z
+ z−5
if 5 < z
1+z 2 1+5−z 2

fz

0.4

0.2

0
2 4 z 6 8 10

-0.2

-0.4
PHY 5346
Homework Set 9 Solutions – Kimel

2. 5.18
a) From the results of Problem 5.17, we can replace the problem stated by the system

μ r −1
where I ∗ is equidistant from the interface and is equal to I ∗ = μ r +1
I. The radius of each current
loop is a. Now from Eq. (5.7)
⃗on I = I ∫ dl⃗ × Br⃗
F

dl⃗ × B ⃗ r + dl⃗ × B
⃗ = dl⃗ × B ⃗ θ = dlB r −θ̂ + dlB θ r̂

By symmetry, only the z − component survives, so, from the figure

dl⃗ × B
⃗ ⋅ ẑ = dlB r a + dlB θ 2d
4d + a 2
2
4d 2 + a 2
So

Fz = 2πaI aB + 2dB 


r θ
4d 2 + a 2

with B r and B θ given by Eqs. (5.48) and (5.49) and cos θ = 2d


,r= 4d 2 + a 2 , and I → I ∗ .
4d 2 +a 2
c) To determine the limiting term, simply let r → 2d and take the lowest non-vanishing term in the
expansion of the magnetic flux density.

F z = πaI aB r + 2dB θ 


d

μ0I∗ a a μ0I∗ a2
F z = πaI a + 2d − 1 − a
d 4d 2d 2 4 2d 3 2d

3πμ 0 a 4 I × I ∗
Fz → −
32 d4
The minus sign shows the force is attractive if I and I ∗ are in the same direction. This same result
can be gotten more directly, using
F z = ∇ z mB z 
with m = πa 2 I, and (from Eq. (5.64))
μ0 2m ∗
Bz =
4π z3
with m ∗ = πa 2 I ∗ , and z = 2d
μ0 3πμ 0 a 4 I × I ∗
Fz = 2πa 2 I ∗ πa 2 I − 3 4 =−
4π 2d 32 d4
with agrees with out previous result.
PHY 5346
Homework Set 10 Solutions – Kimel

4. 5.16 a) The system is shown in the figure

I shall use the magnetic potential approach and will call inside the sphere region 1 and outside the
sphere region 2.

φ 1 = φ loop + > A l r l P l
l

φ 2 = φ loop + > B l r −l−1 P l


l

⃗ φ, and we have the boundary conditions,


⃗ = −∇
where H
H 1|| = H 2|| → φ 1 r = b = φ 2 r = b

μ 0 ∂ φ 1 r = b = μ ∂ φ 2 r = b
∂r ∂r
We are given that b >> a, so

φ loop = 1 m cos θ
4π r 2
with m = πa 2 I. (From the form of φ loop, only the l = 1 term contributes.) The boundary
conditions give
A 1 b 1 = B 1 b −1−1

2μ 0 m 2μm
− 3
+ μ0A1 = − − 2μB 1 b −3
4πb 4πb 3
So
μ − μ 0 
A 1 = − 2 m3
4π b 2μ + μ 0 
On the inside, at the center of the loop

H ⃗ φ loop − ⃗
⃗ = −∇ ∇A 1 r cos θ
⃗ φ loop at the center of the loop, which is directed in the z direction.
From Eq. (5.40), we are given −∇

H z = μ10 −B θ  − A 1

If μ >> μ 0

A 1 → − 1 m3
4π b
and from (5.40), at r = 0
2 3
H z = I + 1 m3 = I + I a 3 = I 1 + a 3
2a 4π b 2a 4 b 2a 2b
PHY 5346
Homework Set 9 Solutions – Kimel

1. 5.10
a) From Eq. (5.35)
′2
μ0 I dr ′ dΩ ′ sin θ ′ cos ϕ ′ δcos θ ′ δr ′ − a
A φ r, θ =
4π a
∫r |x⃗ − ⃗
x′ |
Using the expansion of 1/|x⃗ − ⃗
x ′ | given by Eq. (3.149),


1
|x⃗ − ⃗
x′ |
4
= π ∫ 0 dk coskz − z ′ 1 I 0 kρ < K 0 kρ ∑  + > cosmϕ − ϕ ′ I m kρ < K m kρ ∑ 
2
m=1

We orient the coordinate system so ϕ = 0, and because of the cosϕ ′ factor, m = 1. Thus,
μ 0 I 4π ∞
A φ r, θ =
4π a π
∫ 0 dk ∫ r ′2 dr ′ d cos θ ′ sin θ ′ δcos θ ′ δr ′ − a coskzI 1 kρ < K 1 kρ ∑ 
μ ∞
A φ r, θ = π0 aI ∫ dk coskzI 1 kρ < K 1 kρ ∑ 
0

where ρ < ρ ∑  is the smaller (larger) of a and ρ.

b) From problem 3.16 b),




1
|x⃗ − ⃗
x′ |
= > ∫ 0 dke imφ−φ  J m kρJ m kρ ′ e −k|z|

m=−∞

Note z ′ = 0, and φ = 0, so
μ 0 Ia ∞
Aφ =
2
∫ 0 dke −k|z|J 1 kρJ 1 ka
PHY 5346
Homework Set 10 Solutions – Kimel

3. 5.8 Using the same arguments that lead to Eq. (5.35), we can write

μ0 d 3 x cos φ ′ J φ r ′ , θ ′ 
Aφ =

∫ |x⃗ − ⃗x′ |
Choose ⃗
x in the x − z plane. Then we use the expansion
r l< m∗ ′ ′ m
1
|x⃗ − ⃗
x′ |
= ∑ 2l4π+ 1 r l+1
Y l θ , φ Y l θ, 0
l,m ∑

The cos φ ′ factor leads to only an m = 1 contribution in the expansion. Using

Y ml θ, 0 = 2l + 1 l − m! P m cos θ


4π l + m! l
l−1!
and l+1!
= 1
ll+1
, we have on the inside
′ ′ ′
μ0
∑ ll +1 1 r l P1l cos θ ∫ d 3 x ′ Pl cos θr ′l+1
J φ r , θ 
1
Aφ =
4π l

which can be written


μ0
Aφ = −
4π ∑ m l r l P1l cos θ
l

with
′ ′ ′
∫ d 3 x ′ Pl cos θr ′l+1
J φ r , θ 
1
ml = − 1
ll + 1
A similar expression can be written on the outside by redefining r < and r ∑ .
PHY 5346
Homework Set 10 Solutions – Kimel

2. 5.2 a) The system is described by

First consider a point at the axis of the solenoid at point z 0 . Using the results of problem 5.1,
μ0
dφ m = NIdzΩ

From the figure,

⃗ ρdρ
∫ r̂ ⋅r 2dA ∫ dArcos θ = 2πz ∫
R
Ω= = = 2π − z +1
2 2 3/2
0 ρ + z 
2
R + z 2 
2

μ0 ∞ μ0
φm = NI ∫ z − 1 + 1z dz = NI −z 0 + R 2 + z 20 
2 z0 R + z 
2 2 2

μ0 μ0 −z 0 + R 2 + z 20 
Br = − NI ∂ −z 0 + R 2 + z 20  = NI
2 ∂z 0 2 R 2 + z 20 

In the limit z 0 → 0
μ0
Br = NI
2
By symmetry, thej loops to the left of z 0 give the same contribution, so
B = B l + B r = μ 0 NI

H = NI
⃗ is directed along the z axis, so
By symmetry, B
⃗ =0
⃗⋅B
δφ m = −δρ
⃗ is directed ⊥ to the z axis. Thus for a given z, φ m is independent of ρ, and consequently
if δρ
H = NI
everywhere within the solenoid.
If you are on the outside of the solenoid at position z 0 , by symmetry the magnetic field must be in
the z direction. Thus using the above argument, φ m must not depend on ρ. Set us take ρ far away
from the axis of the solenoid, so that we can replace the loops by elementary dipoles m ⃗ directed along
the z axis. Thus for any point z 0 we will have a contributions
⃗ ⋅ ⃗r 1
m ⃗ ⋅ ⃗r 2
m
φm  3
+
r1 r 32
⃗ ⋅ ⃗r 1 = −m
where m ⃗ ⋅ ⃗r 2 and r 1 = r 2 . Thus

H=0
PHY 5346
Homework Set 10 Solutions – Kimel

1. 5.1 The system is described by

We want to show
μ0I
φm = − Ω

Suppose the observation point is moved by a displacement δx⃗, or equivlently that the loop is
displaced by −δx⃗.
⃗ φ m , then
⃗ = −∇
If we are to have B

δφ m = −δx⃗ ⋅ B
Using the law of Biot and Savart,

μ0I dl⃗′ × ⃗r μ0I δx⃗ × dl⃗′ μ0I δx⃗ × dl⃗′


δφ m = −

∮ δx⃗ ⋅ r3
=−

∮ ⃗r ⋅ r3
=−

∮ r̂ ⋅ r2

δφ m = −
μ0I

∮ r̂ ⋅ δdA
r2
=−
μ0I

δΩ

Or,
μ0I
φm = − Ω

PHY 5346
HW Set 6 Solutions – Kimel

3. 4.10 The system is described by

a) Since there is azimutal symmetry,


Φr, θ = ∑A l r l + B l r −l−1 Pl cos θ
l
Also
⃗ = x⃗E
D ⃗ Φr, θ
⃗ = −x⃗∇

D r = −x⃗ ∑lA l r l−1 − l + 1B l r −l−2 P l cos θ


l

between the spheres,


∫ D r dΩr 2 = Q, and is independent of r.
Thus
x⃗B 0
A l = 0, B l = 0, l ≠ 0 → D r =
r2
∫ D r dΩr 2 = 2πB 0 0 ∫ d cos θ +  ∫ d cos θ
0 1
= 2πB 0  0 +  = Q
−1 0
Q
B0 = 
2π 0 1 + 0 
⃗ =
E
Q


2π 0 1 + 0 r 2
b)
∫ D r dA = D r A = σ f A → σ f = D r = x⃗E r
Q
σf = 
, cos θ ≥ 0
2π 0 1 + 0 r 2
Q
σf = , cos θ < 0
2π1 + 0 r 2
c)
∫ ρ pol dV = σ pol A = ∫ −∇⃗ ⋅ P⃗dV = −PA → σ pol = −P = − 0 χ e E
Q
σ pol = −x⃗/ 0 − 1
2π1 + 0 r 2
Notice
Q
σ pol + σ f = σ tot = =  0 E, as expected.
2π1 + 0 r 2
PHY 5346
HW Set 7 Solutions – Kimel

4. 4.9 a) The system is described by

Since there is azimuthal symmetry, choosing the z-axis through q,

Φ out = 1
4π 0
∑ B l r −l−1 Pl + q
|x⃗ − ⃗
x′ |
l

∑ B l r −l−1 Pl + q
∑ r< l
Φ out = 1 Pl
4π 0 r∑ r∑
l l

∑ A l r l Pl + q
∑ r< l
Φ in = 1 Pl
4π 0 r∑ r∑
l l
Boundary conditions: At the surface, r ′ = d = r ∑ , r = a = r < .

1) Φ out = Φ in |r=a , or
B l = A l a 2l+1
2)  ∂r∂ Φ in = ∂r∂ Φ out |r=a , or letting k = 0

k ∑ lA l a l−1 Pl + q a l−1
d
l
dl
Pl = ∑ −l + 1B l a −l−2 Pl + q a l−1
d
l
dl
Pl
l l

= ∑ −l + 1A l a l−1 Pl + q a l−1


d
l
dl
Pl
l
or
a1 − kl
Al =
1 + kl + 1d l+1
a1 − kla 2l+1
Bl =
1 + kl + 1d l+1
Remember that P l = 2+1 4π
Y 0l , and substitute the above coefficients into the expansion to get the
answer requested by the problem.
PHY 5346
HW Set 7 Solutions – Kimel

3. 4.7 a) Since ρ does not depend on φ, we can write it in terms of spherical harmonics with
m = 0. First note
Y 02 = 5 3 1 − sin 2 θ − 1
4π 2 2
or
sin 2 θ = − 2 4π Y 02 + 4π 2 Y 00
3 5 3
Thus only the m = 0, l = 0, 2 multipoles contribute.
2 4π ∞ 2 1 2 −r
q 00 =
3
∫ 0 r 64π r e dr = 2 34π 8π 3 = 1
2 π

q 20 = − 2
3

5
∫0 r4 1 r 2 e −r dr = − 2
64π 3
4π 45 = −3 5
5 4π π
1 Y0 Y0 1 P 4π q P 2
≠x⃗ = 4πq 00 r0 + 4πq 20 23 = 4π q 00 r0 +
4π 0 5r 4π 0 5 20 r 3

≠x⃗ = 1 P0 − 6 P2
4π 0 r r3
b)
⃗ d x
′ 3 ′
≠x⃗ = 1
4π 0
∫ ρx
|x⃗ − ⃗
x′ |
Using
= 4π ∑
1 1 r< l
′ ′
l θ , φ Y l θ, φ
Y m∗ m
|x⃗ − ⃗
x′ | lm
2l + 1r ∑ r∑
, we see only the l = 0, 2 and m = 0 terms of the expansion contribute in the potential. Next take
r ′ ∑ r.
′ ρx⃗′  ′
≠x⃗ = 1 4π ∑ 1 r l Y ml θ, φ ∫ Y m∗ ′ ′ ′2
l θ , φ r dΩ dr
4π 0 lm
2l + 1 r ′l+1

∞ 0 ∞
≠x⃗ = 1 4π Y 0 4π 2
4π 0 0
3
∫0 1 r 2 e −r rdr + Y 2 r 2 − 2
64π 5 3

5
∫0 1 r 2 e −r 1 dr
64π r
0
≠x⃗ = 1 4π Y 0 4π 2 3 + Y 2 r 2 − 2 4π 1
0
4π 0 3 32π 5 3 5 64π

≠x⃗ = 1 4π P 2 3 + P 2 r 2 − 2 1 = 1 P0 − r 2 P2
0
4π 0 3 32π 5 3 64π 4π 0 4 120
PHY 5346
HW Set 7 Solutions – Kimel

2. 4.2 We want to show that we can obtain the potential and potential energy of an elementary
diplole:

p⋅⃗x
Φx⃗ = 1
4π 0 r 3
W = −p ⃗⋅E ⃗ 0
from the general formulas
⃗ d x
′ 3 ′
Φx⃗ = 1
4π 0
∫ ρx
|x⃗ − ⃗
x′ |
W= ∫ ρx⃗Φx⃗d 3 x
using the effective charge density
⃗⋅⃗
ρ eff = −p ∇δx⃗
where I’ve chosen the origin to be at ⃗x0.
−p ⃗
⃗ ⋅ ∇ δx⃗′ d 3 x ′

Φx⃗ = 1 ∫ =− 1 ⃗ p ⋅ ∫⃗
∇ 1 δx⃗′ d 3 x ′
4π 0 |x⃗ − ⃗
x′ | 4π 0 |x⃗ − ⃗
x′ |

Φx⃗ = 1 ⃗ p⋅⃗ x
4π 0 r 3

Similarly,
W= ∫ ρx⃗Φx⃗d 3 x = − ∫ ⃗p ⋅ ⃗∇δx⃗Φx⃗d 3 x = ⃗p ⋅ ∫ δx⃗∇⃗ Φx⃗d 3 x = −p⃗ ⋅ E⃗0
PHY 5346
HW Set 6 Solutions – Kimel

3. 4.1

q lm = ∫ r l Y m∗ ⃗d 3 x = ∑ q i r ll Y m∗
l θ, φρx l θ i , φ i 
i
Using
2l + 1l − m! m
l θ, φ =
Y m∗ P l xe −−mφ = N ml P ml xe −−mφ
4πl + m!
From the figure we get
q lm = a l N ml P ml 0q1 − −1 m 1 − i m  = 0, for m even, so m = 2n + 1, n = 0, 1, 2, . . .
q lm = 2qa l N ml P ml 01 − −1 n i
b) The figure for this system is

Since the sum of the charges equals zero, l ≥ 1.


l x = 1, φ + Y l x = −1, φ = qa N l P l 1 + P l −1
q lm = qa l Y m∗ m∗ l m m m

From the Rodrigues formula for P ml x, we see P ml ±1 = 0, for m ≠ 0. So


q lm = qa l N 0l 1 + −1 l P l 1
Thus l is even, but l ≠ 0
q lm = 2qa l N 0l
c) Using the fact that N 0l = 2l+1

and Y 0l = 2l+1

Pl

x 2qa 2
Φx⃗ = ∑2qa l  Prll+1 ≈
r3
P 2 x = 0 on x-y plane)
l=2
2
qa
Φx⃗ = − 3
r
Let us plot Φx⃗/−q/a, ie, 1
= 1
 ar  3 x3

0 0.5
1 1.5 x 2 2.5 3

The exact answer on the x-y plane is

−q 2 − 2 −q 1 3 5 7 9
Φx⃗ = a x = a x − 3 1
x + 5 1
x − 35 1
x +. . . .
x 1+ 1 4 8 64
x2

So let’s plot 1
x3
, 2
x − 2
1
x 1+
x2

0.3

0.25

0.2

0.15

0.1

0.05

0 1
2 3
x 4 5

where the smaller is the exact answer.


PHY 5346
HW Set 6 Solutions – Kimel

2. 3.10 This problem is described by

a) From the class notes


Φρ, z, φ = ∑A nν sin νφ + B nν cos νφI ν nπρ
L
sin nπz
L

where
∫0 ∫0
L 2π
A nν = 2 1 Vφ, z sin nπa  sinνφdφdz, ν>0
πL I ν nπb L
L

∫0 ∫0
L 2π
B nν = 2 1 Vφ, z sin nπa  cosνφdφdz, ν≠0
πL I ν nπb L
L

∫0 ∫0
L 2π
B nν = 1 1 Vφ, z sin nπa dφdz, ν=0
πL I ν nπb L
L
Noting
π 3π

∫− 2

π
sin νφdφ − ∫ π sin νφdφ
2
=0
2 2

we conclude A nν = 0. Similarly, noting


π 3π
4−1 m
∫−2

π
cos νφdφ − ∫ π cos νφdφ
2
=
2m + 1
, m = 0, 1, 2, . . . .
2 2

where I’ve recognized that ν must be odd, ie, ν = 2m + 1. Also


∫ 0 sin nπz
L
dz = 2 , l = 0, 1, 2, . . . . .
L 2l + 1π
where again I’ve recognized that n must be odd, ie, n = 2l + 1. Thus
16−1 m V
B nν = 2
π I 2m+1  L 2l + 12m + 1
nπb

b) Now z = L/2, L >> b, L >> ρ. Then from the class notes


2l + 1πρ 1 2l + 1πρ m+1
I 2m+1  
L Γ2m + 2 2L
Also
2l + 1π
sin = −1 l
L
so
l+m ρ 2m+1
Φρ, z, φ = ∑ π 2 2l16−1 V
+ 12m + 1 b
cos2m + 1φ
l,m

Using

∑ 1lx + 1 −1 l
2l+1
−1x
tan =
l=0

∑ 2l−1
l
π = tan −1 1 =
4 l=0
+1
so
−1 m ρ 2m+1
Φρ, z, φ = 4V
π ∑ 2m +1 b
cos2m + 1φ
m
Remembering from problem 2.13 that
ρ
−1 m ρ 2m+1 2 cos φ
∑ 2m + 1 b
cos2m + 1φ = 1 tan −1
2
b
ρ2
m 1− b2

we find
ρ
2 cos φ
Φρ, z, φ = 2V
π tan
−1 b
ρ2
1− b2
which is the answer for problem 2.13.
PHY 5346
HW Set 5 Solutions – Kimel

4. 3.4 Slice the sphere equally by n planes slicing through the z axis, subtending angle Δφ about
this axis with the surface of each slice of the pie alternating as ±V.
φr, θ, φ = > A lm r l Y ml θ, φ
l,m

so
A lm = 1l
a
∫ dΩY ml θ, φ ∗ φa, θ, φ

Symmetries:
A l−m = −1 m A lm  ∗
φr, θ, φ + 2Δφ = φr, θ, φ
where
Δφ = 2π
2n
Thus
m = ±n, and integral multiples thereof
φ−r⃗ = −φr⃗, n = 1
φ−r⃗ = φr⃗, n > 1
Since
PY ml θ, φ = −1 l Y ml θ, φ
Then
l is odd for n = 1; l is even for n > 1
Thus we only have contributions of l ≥ n. Using
A lm = 1l ∫ dΩY ml θ, φ ∗ φa, θ, φ
a
The integral over φ can be done trivially, since the integrand is just e −imφ leaving the desired answer
in terms of an integral over cosθ.

n = 1 case: I am going to keep only the lowest novanishing terms, involving A 11 and A 1−1 .

1 ∗
φ = rA 11 Y 11 + A 1−1 Y −1
1  = rA 11 Y 1 + A 11 Y 1   = 2r ReA 11 Y 1 
1 1

Y 11 = − 3 1 − x 2  1/2 e iφ

π
∫ −1 1 − x 2  1/2 dx ∫ 0 e −iφ dφ − ∫ π
1 2π
A 11 = − 1a 3 V e −iφ dφ

A 11 = 2iπ
a
3 V

φ = 2r Re 2iπ 3 V − 3 sin θe iφ = 3r Vsin θ sin φ


a 8π 8π 2a
From the figure

we see
sin θ sin φ = cos θ ′
So
φ = 3r Vcos θ ′ = V 3 ar P 1 cos θ ′  +. . . . . .
2a 2
The other terms, for l = 2, 3, can be obtained in the same way in agreement with the result of
(3.36)
PHY 5346
HW Set 5 Solutions – Kimel

3. 3.1 The system is pictured in the following figure:

1
∫ 0 Pl xdx
The problem is symmetric around the z axis so
φr, θ = ∑A l r l + B l r −l−1 Pl cos θ
l
The A l and B l are determined by the conditions
1)
∫ −1 φa, xPl xdx =
1
2 A a l + B l a −l−1 
2l + 1 l
2)
∫ −1 φb, xPl xdx =
1
2 A b l + B l b −l−1 
2l + 1 l
Solving these two equations gives
2l + 1 a l+1 ∫ φa, xP l xdx − b l+1 ∫ φb, xP l xdx
1 1
Al =
2a 2l+1
−b 2l+1 −1 −1

B l = a l+1 2l + 1 ∫ −1 φa, xPl xdx − A l a 2l+1


1

2
Using
∫ −1 φa, xPl xdx = V ∫ 0 Pl xdx
1 1

∫ −1 φb, xPl xdx = V ∫ −1 Pl xdx = V−1 l ∫ 0 Pl xdx


1 0 1

So
2l + 1 ∫
1
Al = Va l+1
− b l+1
−1 l
 P l xdx
2a 2l+1 − b 2l+1  0

B l = a l+1 2l + 1 V ∫ P l xdx − A l a 2l+1


1

2 0
Note that
∫ 0 Pl xdx =
1 ∫ 1 P l xdx
1

2 −1
1
for l even. For even l > 0, ∫ P l xdx = 0. Thus we have
0

∫ 0 P0 xdx = 1; ∫ 0 P1 xdx = ∫ 0 P3 xdx = − 18


1 1 1
1,
2
and
A0 = V , A1 = 3 Va 2 + b 2 , A 3 = − 7 Va 4 + b 4 
2 4a 3 − b 3  16a 7 − b 7 

B 0 = 1 Va − 1 Va = 0
2 2
B1 = 3 a2V − 3 Va 2 + b 2 a 3 = 3 Va 2 b 2 b3 + a 3
4 4a 3 − b 3  4 −a + b

= − 7 Va 4 b 4 b 7 + a 7
3 3
B3 = − 7 a4V − a7 − 7 Va 4 + b 4 
16 16a 7 − b 7  16 −a + b
1
As b → ∞, only the B l terms (and A 0  survive. Thus using the general expression for ∫ P l xdx
0
given by (3.26)
3 4
φr, θ = V P 0 x + 3 a2 P 1 x − 7 a4 P 3 x +. . . . .
2 2 r 8 r
Let’s now solve the problem neglecting the outer sphere (since b → ∞ using the Green’s function
result
(2.19) this integral give, for cosθ = 1

φr, θ = V 1 − ρ 2  1 − 1
2 1 − ρ 1 + ρ 2 
with ρ = a/r. Expanding the above,
2 4
φr, θ = V ar + 3 a2 − 7 a4 +. . . . . .
2 2 r 8 r
Comparing with our previous solution with x = 1, we see the Green’s function solution differs by
having a B 0 term and by not having an A 0 term. All the other higher power terms agree in the series.
This difference is due to having a potential at ∞ in the original problem.
PHY 5346
HW Set 5 Solutions – Kimel

2. 2.23 The system is pictured in the following figure:

a) As suggested in the text and in class, we will superpose solutions of the form (2.56) for the two
sides with Vx, y, z = V.
1) First consider the side Vx, y, a = z :

Φ 1 x, y, z = ∑ A nm sinα n x sinβ m y sinhγ nm z
n,m=1
π
with α n = nπ
a , βm = mπ
a , γ nm = a n 2 + m 2 . Projecting out A nm using the orthogonality of the
sine functions,
A nm = 16V
sinhγ nm anmπ 2
where both n, and m are odd. (Later we will use n = 2p + 1, m = 2q + 1
2) In order to express Φ 2 x, y, z in a form like the above, we make the coordinate transformation
x ′ = y, y ′ = x, z ′ = −z + a
So
Φ 2 x, y, z = Φ 1 x ′ , y ′ , z ′  = Φ 1 y, x, = z + a
Φx, y, z = Φ 1 x, y, z + Φ 2 x, y, z
b)

−1 p+q
Φ a , a , a  = 16 ⋅2 V ∑
2 2 2 π p,q=0 2p + 12q + 1 cosh γ nm a
2

where I have used the identity


γ nm a γ a
sinhγ nm a = 2 sinh  cosh nm 
2 2
∞ −1 p+q
Let fp, q ≡ ∑ p,q=0 π
2p+12q+1 cosh 2p+1 2 +2q+1 2 2
(p,q) fp, q Error Sum
0,0 0.213484 4.4% .214384
1,0 −0. 004641 2.13% 0.20974
0,1 −0. 004641 0.013% 0.20510
1,1 0.0002835 0.015% 0.20539
The first three terms give an accuracy of 3 significant figures.

Φ a , a , a  = 16 ⋅ 0. 20539 V = 0. 332 96V


2 2 2 π2
Φ av  a , a , a  = 2 V = 0. 333. . . . V
2 2 2 6
c)
σx, y, a = − 0 ∂ Φ|z=a
∂z
σx, y, a = − 162 0 V
π

coshγ nm a − 1
= − 162 0 V ∑ sinα n x sinβ m y
π n,m odd
sinhγ nm a

γ nm a
σx, y, a = − 162 0 V ∑ sinα n x sinβ m y tanh
π n,m odd
2
PHY 5346
HW Set 5 Solutions – Kimel

1. 2.13 The system is pictured in the following figure:

a) Notice from the figure, Φρ, −φ = Φρ, φ; thus from Eq. (2.71) in the text,

Φρ, φ = a 0 + ∑ a n ρ n cosnφ
n=1

V1 + V2
∫ −π/2 Φb, φ = 2πa 0 = πV1 + πV2 → a 0 =
3π/2

2
Using
∫ −π/2 cos mφ cos nφdφ = δ nm π
3π/2

Applying this to Φ, only odd terms m contribute in the sum and


2V 1 − V 2  m−1
am = −1 2
πmb m
Thus
2V 1 − V 2  i m ρ m e imφ
Φρ, φ = V 1 + V 2 + π Im ∑
2 mb m
m odd

Using
2 ∑ x m = ln 1 + x
m 1 − x
m odd

and
Im lnA + iB = tan −1 B/A
we get
ρ
V + V2 V 1 − V 2  2 b cos φ
Φρ, φ = 1 + π tan −1 ρ2
2 1− b2
as desired.
b)

σ = − 0 ∂ Φρ, φ|ρ=b
∂ρ
ρ
V 1 − V 2  ∂ 2 b cos φ
σ = − 0 π tan −1
∂ρ 1−
ρ2
b2 ρ=b

V1 − V2 b +ρ 2 2
V − V2
σ = −2 0 π bcos φ 4 |ρ=b = − 0 1
b − 2b ρ + ρ + 4ρ b cos φ
2 2 4 2 2 2 πb cos φ
:
PHY 5346
HW Set 4 Solutions – Kimel

4. 2.11 The system is pictured in the following figure:

a) The potential for a line charge is (see problem 2.3)


φr⃗ = λ ln rr∞
2π 0
Thus for this system

φr⃗ = τ ln r∞ + τ ′ ln r∞
2π 0 ⃗
⃗r − R 2π 0 ⃗′
⃗r − R
To determine τ ′ and R ′ , we need two conditions:

I) As r → ∞, we want φ → 0, so τ ′ = −τ.

II) φr⃗ = b  = φr⃗ = −b  or


′ ′
ln b − R = ln b + R
R−b b+R
or
b − R′ = b + R′
R−b b+R

This is an equation for R with the solution
2
R′ = b
R
The same condition we found for a sphere.
b)
b4 2
r2 + − 2r bR cos φ
φr⃗ = τ ln R2
4π 0 r 2 + R 2 − 2rR cos φ
as r → ∞
τ ln 1 − 2b cos φ/rR
2
φr⃗ = = τ ln 1 − 2 b 2 − R 2  cos φ
4π 0 1 − 2R cos φ/r 4π 0 rR
Using
ln1 + x = x − 1 x 2 + 1 x 3 + Ox 4 
2 3
φr⃗ = − τ 1 b 2 − R 2  cos φ
2π 0 rR
c)
b4 2
r2 + − 2r bR cos φ
σ = − 0 ∂ φ|r=b = − τ ∂ ln R2
∂r 4π ∂r r 2 + R 2 − 2rR cos φ
r=b

τ 1−y 2
σ=
2πb y + 1 − 2y cos φ
2

τ 1−y 2
where y = R/b. Plotting σ/ 2πb
= y 2 +1−2y cos φ
, for y = 2, 4, gives
1 − y2
gy =
y 2 + 1 − 2y cos φ
g2, g4 = − 3
5−4 cos φ
, − 17−815cos φ

0.5 1 1.5 2 2.5 3


0

-0.5

-1

-1.5

-2

-2.5

-3

d) If the line charges are a distance d apart, then the electric field at τ from τ ′ is, using Gauss’s law

E= τ
2π 0 d
The force on τ is τLE, ie,

F = ττ L = − τ L , and the force is attractive.
2

2π 0 d 2π 0 d
PHY 5346
HW Set 4 Solutions – Kimel

3. 2.10 As done in class we simulate the electric field E 0 by two charges at ∞

σ = 3 0 E 0 cos θ
a) This charge distribution simulates the given system for cos θ > 0. We have treated this probem
in class. The potential is given by
3
φx⃗ = −E 0 1 − a3 r cos θ
r
Using
σ = − 0 ∂ φ|surface
∂n
We have the charge density on the plate to be
σ plate = − 0 ∂ φ|z=0 =  0 E 0 1 − a 3
3

∂z ρ
σ plate
For purposes of plotting, consider  0 E 0 = 1 − x 3
1

0.8

0.6

0.4

0.2

0
2 4 x 6 8 10

σ boss = 3 0 E 0 cos θ =
σ boss
For plotting, we use 3 0 E 0
= cos θ
1

0.8

0.6

0.4

0.2

0
0.2 0.4 0.6 0.8 1 1.2 1.4

b)
q = 3 0 E 0 2πa 2 ∫ xdx = 3π 0 E 0 a 2
1

0
c) Now we have

1 q q′ q q′
φr⃗ = + − −
4π 0 ⃗r − ⃗
d ⃗r − ⃗
d′ ⃗r + ⃗
d ⃗r + ⃗
d′
where q ′ = −q ad , d ′ = a2
d
.

−q d 2 − a 2  d 2 − a 2 
σ = − 0 ∂ φ|r=a = −
∂r 4π a⃗a−⃗ d
3
a⃗a+⃗ d
3

−q d 2 − a 2  d 2 − a 2 
∫0
1
q ind = 2πa 2 − dx
a−⃗ a+⃗
4π 3 3
a⃗ d a⃗ d

−qa 2 d 2 − a 2  1 1 − 1
q ind = + 1 − 1
2a da d−a a +d
2 2 d+a a + d2
2

d 2 − a 2 
q ind = −1 q d − a
2 2
2d − 2 = −q 1 −
2 d d2 − a2 a + d2
2
d a2 + d2
PHY 5346
HW Set 4 Solutions – Kimel

2. 2.9 The system is pictured below

a) We have treated this probem in class. We found the charge density induced was
σ = 3 0 E 0 cos θ
We also note the radial force/unit area outward from the surface is σ 2 /2 0 . Thus the force on the
right hand hemisphere is, using x = cos θ
⃗ = 1 3 0 E 0  2 2πR 2 ∫ x 3 dx = 1 3 0 E 0  2 2πR 2 /4 = 9 π 0 E 20 R 2
F z = 1 ∫ σ 2 ẑ ⋅ da
1

2 0 2 0 0 2 0 4
An equal force acting in the opposite direction would be required to keep the hemispheres from
sparating.

b) Now the charge density is


Q Q Q
σ = 3 0 E 0 cos θ + 2
= 3 0 E 0 x + 2
= 3 0 E 0 x +
4πR 4πR 12π 0 E 0 R 2
2
Fz = 1
2 0
∫ σ 2 ẑ ⋅ da⃗ = 1 3 E  2 2πR 2 ∫ 1 x x +
2 0 0 0
0
Q
12π 0 E 0 R 2
dx

Thus
F z = 9 π 0 E 20 R 2 + 1 QE 0 + 1 Q2
4 2 32 0 πR 2
An equal force acting in the opposite direction would be required to keep the himispheres from
separating.
PHY 5346
HW Set 4 Solutions – Kimel

1. 2.8 The system is pictured below

a) Using the known potential for a line charge, the two line charges above give the potential

φr⃗ = 1 λ ln rr = V, a constant. Let us define V ′ = 4π 0 V
2π 0
Then the above equation can be written
r ′ 2 = e Vλ′ or r ′2 = r 2 e Vλ′
r
⃗ , the above can be written
Writing r ′2 = ⃗r − R
2

2
V′
⃗r + ẑ R = R2e λ
V′ V′
e λ − 1 e λ − 1 2
V′
The equation is that of a circle whose center is at −ẑ R
V′
, and whose radius is a = Re 2λ
V′
e λ −1 e λ −1
b) The geometry of the system is shown in the fugure.

Note that
d = R + d1 + d2
with
d1 = R , d2 = R
V ′a −V ′b
−1 −1
e λ e λ

and

Va −V ′b

a = ReV ′a

, b = Re
−V ′b

e λ − 1 e λ − 1
Forming
2 2 2

Va −V ′b

d −a −b =
2 2 2
R+ R + R − Re 2λ − Re 2λ
V ′a −V ′b V ′a −V ′b
−1
e λ e λ −1 e λ − 1 e λ − 1
or
V ′a −V ′b
R2 e λ +1
d2 − a2 − b2 = −V ′b
V ′a
e λ − 1e λ − 1
Thus we can write
V ′a −V ′b
e λ +1 V ′a −V ′b − V ′a −V ′b
d2 − a2 − b2 = +e V ′a − V ′b
−V ′b
= e 2λ 2λ
= cosh
2ab V ′a 2 2λ
2e 2λ e 2λ

or
V a − V b = 1 cosh −1 d 2 − a 2 − b 2
λ 2π 0 2ab
Capacitance/unit length = C =
Q/L
= λ = 2π 0
L Va − Vb Va − Vb cosh −1 d 2 −a 2 −b 2
2ab

c) Suppose a << d , and b << d , and a = ab , then
2 2 2 2

C = 2π 0 2π 0
= 2 1−a 2 +b 2 /d 2
L −1 d 2 −a 2 −b 2 −1 d
cosh ′2 cosh
2a 2a ′2

d 2 1 − a 2 + b 2 /d 2 
cosh −1 = 2π 0 L
2a ′2 C
d 2 1 − a 2 + b 2 /d 2 
2π 0 L

= e C
+ negligible terms if 2π 0 L >> 1
2a ′2 2 C
or
d 2 1 − a 2 + b 2 /d 2 
ln = 2π 0 L
a ′2 C
or
C = 2π 0
L d 2 1−a 2 +b 2 /d 2
ln a ′2

Let us defind α 2 = a 2 + b 2 /d 2 , then


C = 2π 0 = 2π  0d 2 + 2π 2 0d 2 α 2 + Oα 4 
L d 2 1−α 2
ln a ′2 ln a ′2
ln a ′2

The first term of this result agree with problem 1.7, and the second term gives the appropriate
correction asked for.
d) In this case, we must take the opposite sign for d 2 − a 2 − b 2 , since a 2 + b 2 > d 2 . Thus
C = 2π 0
L −1 a 2 +b 2 −d 2
cosh 2a ′2

If we use the identiy, lnx + x 2 − 1  = cosh −1 x, G.&R., p. 50., then for d = 0
C = 2π 0 2π 0
a 2 +b 2 2 −b 2 =
L ln + a2ab ln ab
2ab
in agreement with problem 1.6.
PHY 5346
HW Set 3 Solutions – Kimel

5. 2.7 The system is described by

a) The Green’s function, which vanishes on the surface is obviously

Gx⃗, ⃗
x′ = 1 − 1
|x⃗ − ⃗
x′ | |x⃗ − ⃗
x ′I |
where
x ′ = x ′ î + y ′ ̂ + z ′ k̂ , ⃗
⃗ x ′I = x ′ î + y ′ ̂ − z ′ k̂
b) There is no free charge distribution, so the potential everywhere is determined by the potential
on the surface. From Eq. (1.44)
∂Gx⃗, ⃗ x′ ′
φx⃗ = − 1 ∫ φx⃗′  ′ da
4π S ∂n
Note that n̂ ′ is in the −z direction, so
∂ Gx⃗, ⃗x ′ |z ′ =0 = − ∂ Gx⃗, ⃗ x ′ |z ′ =0 = − 2z
∂n ′ ∂z ′ 2
x − x  + y − y ′  2 + z 2
3/2

So
ρ ′ dρ ′ dφ ′
φx⃗ = z V ∫ ∫
a 2π

2π 3/2
x − x ′  + y − y ′  2 + z 2
0 0 2

where x ′ =ρ ′ cos φ ′ , y ′ = ρ ′ sin φ ′ .


c) If ρ = 0, or equivalently x = y = 0,
ρ ′ dρ ′ dφ ′ ρdρ
φz = z V ∫ ∫ ∫
a 2π a
= zV
2π ′2
0 0 ρ + z  2 3/2 0 ρ + z 2 
2 3/2

z − a 2 + z 2  z
φz = zV − = V 1−
z a + z 
2 2
a + z 2 
2

d)
ρ ′ dρ ′ dφ ′
φx⃗ = z V ∫ ∫
a 2π

2π 3/2
0 0 ⃗−ρ
ρ ⃗ ′ 2 + z2
⃗, then ρ
In the integration choose the x −axis parallel to ρ ⃗⋅ρ ⃗ ′ = ρρ ′ cos φ ′
ρ ′ dρ ′ dφ ′
φx⃗ = z V ∫ ∫
a 2π

2π 0 0 ρ 2 + ρ ′2 − 2ρ ⃗⋅ρ ⃗ ′ + z 2  3/2
Let r 2 = ρ 2 + z 2 , so
ρ ′ dρ ′ dφ ′
φx⃗ = z V3 ∫ ∫
a 2π

2π r 0 0 ρ ′2 −2ρ ⃗′
⃗⋅ρ 3/2
1+ r2
We expand the denominator up to factors of O1/r , ( and change notation
4

φ ′ → θ, ρ ′ → α, r 2 → β12 

∫0 ∫0
a 2π
φx⃗ = z V3 αdαdθ
2π r α 2 −2ρ ⃗⋅α
⃗ 3/2
1+ r2
where the denominator in this notation is written

1
3/2
1 + β α − 2ρα cos θ
2 2

or, after expanding,

φx⃗ = z V3 ∫ αdα ∫ 1 − 3 β 2 α 2 + 3β 2 ρα cos θ + 15 β 4 α 4 − 15 β 4 α 3 ρ cos θ + 15 β 4 ρ 2 α 2 cos 2 θ dθ


a 2π

2π r 0 0 2 8 2 2
Integrating over θ gives
φx⃗ = z V3 ∫ α 2π + 15 β 4 α 4 π − 3β 2 α 2 π + 15 β 4 ρ 2 α 2 π dα
a

2π r 0 4 2
Integrating over α yields
φx⃗ = z V3 5 β 4 πa 6 − 3 a 4 β 2 π + 15 a 4 β 4 ρ 2 π + πa 2
2π r 8 4 8
or
Va 2 a2 a 4 + 3ρ 2 a 2
φx⃗ = 1− 3 + 5
2ρ + z 2  3/2
2 4 ρ + z 2 
2 8 ρ 2 + z 2 
2
PHY 5346
HW Set 3 Solutions – Kimel

4. 2.6 We are considering two conducting spheres of radii r a and r b respectively. The charges on
the spheres are Q a and Q b .
a) The process is that you start with q a 1 and q b 1 at the centers of the spheres, and sphere a
then is an equipotential from charge q a 1 but not from q b 1 and vice versa. To correct this we use
the method of images for spheres as discussed in class. This gives the iterative equations given in the
text.

b) q a 1 and q b 1 are determined from the two requirements


∞ ∞
∑ q a j = Q a and ∑ q b j = Q b
j=1 j=1

As a program equation, we use a do-loop of the form


n
−r a q b j − 1
∑q a j = d b j − 1

j=2

and similar equations for q b j, x a j, x b j, d a j, d b j. The potential outside the spheres is given
by
n n
q a j q b j
φx⃗ = 1
4π 0
∑ ⃗ ̂
x − x a jk
+∑
x − d b jk̂

j=1 j=1

This potential is constant on the surface of the spheres by construction.


And the force between the spheres is
q a jq b k
F= 1 ∑
4π 0 j,k d − x a j − x b k 2
c) Now we take the special case Q a = Q b , r a = r b = R, d = 2R. Then we find, using the iteration
equations
x a j = x b j = xj
j − 1
x1 = 0, x2 = R/2, x3 = 2R/3, or xj = R
j
q a j = q b j = qj
−1 j+1
qj = q, q2 = −q/2, q3 = q/3, or qj = q
j
So, as n → ∞
∞ ∞
−1 j+1
∑ qj = q ∑ j
= q ln 2 = Q → q =
Q
ln 2
j=1 j=1

The force between the spheres is


−1 j+k
∑ −1
j+k
q2 2
F= 1 ∑
4π 0 R 2 j,k jk 2 − j−1 − k−1 2
= 1 q
4π 0 R 2
jk
j + k 2
j,k
j k
Evaluating the sum numerically
2 2
F= 1 q 0. 0739 = 1 Q 1 0. 0739
4π 0 R 2 4π 0 R ln 2 2
2

Comparing this to the force between the charges located at the centers of the spheres
Q2
Fp = 1
4π 0 R 2 4
Comparing the two results, we see
F = 4 1 2 0. 0739F p = 0. 615F p
ln 2
On the surface of the sphere
∞ ∞
φ= 1
4π 0 ∑ qj
R − xj
=
q
4π 0 R
∑−1 j+1
j=1 j=1

Notice 1 =
1+1 ∑−1 j+1
j=1

1 q Q Q
So φ = = 1 = → C = 2 ln 2 = 1. 386
4π 0 2R 4π 0 2 ln 2R C 4π 0 R
PHY 5346
HW Set 3 Solutions – Kimel

3. 2.5
a)

∞ q2a ∞ q2a
W= ∫ r |F|dy = 4π 0
∫r dy
a2
2
=
8π 0 r 2 − a 2 
y3 1 − y2
Let us compare this to disassemble the charges
qiqj aq 2 q2a
− W′ = − 1
8π 0
> |x⃗i −x⃗j |
= 1
4π 0 r
1
a2
=
4π 0 r 2 − a 2 
>W
i≠j r 1− r2
The reason for this difference is that in the first expression W, the image charge is moving and
changing size, whereas in the second, whereas in the second, they don’t.
b) In this case
∞ ∞ ∞ 2y 2 − a 2 
W= ∫r |F|dy =
q
4π 0
∫r Qdy
y2
− qa 3
∫ r 2 2 2 dy
yy − a 
Using standard integrals, this gives
1 q2a q2a qQ
W= − − r
4π 0 2r − a 
2 2
2r 2

On the other hand


1 aq 2 qQ + ar q aq 2 q2a qQ
− W′ = − r = 1 − 2 − r
4π 0 r − a 
2 2 4π 0 r − a 
2 2
r
The first two terms are larger than those found in W for the same reason as found in a), whereas the
last term is the same, because Q is fixed on the sphere.
PHY 5346
HW Set 3 Solutions – Kimel

2. 2.3 The system is described by

a) Given the potenial for a line charge in the problem, we write down the solution from the figure,

φT = λ ln R2 − ln R2 − ln R2 + ln R2
x⃗ − ⃗ x⃗ − ⃗ x⃗ − ⃗ x⃗ − ⃗
2 2 2
4π 0 xo  x o1  x o2  x o3  2
Looking at the figure when y = 0, x⃗ − ⃗ x o  2 = x⃗ − ⃗ x o1  2 , x⃗ − ⃗ x o2  2 = x⃗ − ⃗x o3  2 , so φ T |y=0 = 0
Similarly, when x = 0, x⃗ − ⃗ x o  = x⃗ − ⃗
2
x o2  , x⃗ − ⃗
2
x o1  = x⃗ − ⃗
2 2
x o3  , so φ T |x=0 = 0
On the surface φ T = 0, so δφ T = 0, however,
∂φ T ∂φ T
δφ T = δx = 0 → = 0 → Et = 0
∂x t t ∂x t
b) We remember
∂φ
σ = − 0 T = −λ
y0 y0
π −
∂y x − x 0  + y 0
2 2
x + x 0  2 + y 20
where I’ve applied the symmetries derived in a). Let

σ/λ = −1
y0 y0
π x − x  2 + y 2 − x + x  2 + y 2
0 0 0 0
This is an easy function to plot for various combinations of the position of the original line charge
(x 0 , y 0 .

c) If we integrate over a strip of width Δz, we find, where we use the integral
x0
∫ 0 x ∓ x 1 2 + y 2 dx = 12 π ± 2 arctan

y0
y0
0 0
∞ ΔQ ∞
ΔQ = ∫ σdxΔz → = ∫ σdx = −2λ π tan
−1 x 0
0 Δz 0
y0
and the total charge induced on the plane is −∞, as expected.
d)
Expanding
ln R2 − ln R2 − ln R2 + ln R2
x − x 0  2 + y − y 0  2 x − x 0  2 + y + y 0  2 x + x 0  2 + y − y 0  2 x + x 0  2 + y + y
to lowest non-vanishing order in x 0 , y 0 gives
xy
16 2 y x
2 0 0
x + y 2 
so
4λ xy
φ → φ asym = π y x
2 0 0
0
x + y 2 
2

This is the quadrupole contribution.


PHY 5346
HW Set 2 Solutions – Kimel

5. 2.2 The system is described by

a) Using the method of images


1 q q′
φx⃗ = +
4π 0 |x⃗ − ⃗
y| |x⃗ − ⃗y′ |
with y ′ = ay , and q ′ = −q ay
2

b) σ = − 0 ∂n∂ φ|x=a = + 0 ∂x∂ φ|x=a


1 ∂ q q′
σ = 0 +
4π 0 ∂x x 2 + y 2 − 2xy cos γ
1/2
x 2 + y ′2 − 2xy ′ cos γ
1/2

y2
a 1−
σ = −q 1 a2
4π y 2 + a 2 − 2ay cos γ 3/2
Note
y2
q induced = a 2 ∫ σdΩ = −q 1 a 2 2πa 1 − 2 ∫ −1
1
dx , where x = cos γ
4π 3/2
a y 2 + a 2 − 2ayx
q 2
q induced = − aa 2 − y 2  = −q
2 aa 2 − y 2 
c)
qq ′ 1 q 2 ay
|F| = = , the force is attractive, to the right.
4π 0 y ′ − y 2 4π 0 a 2 − y 2 
d) If the conductor were fixed at a different potential, or equivalently if extra charge were put on
the conductor, then the potential would be
q q′
φx⃗ = 1 + +V
4π 0 |x⃗ − ⃗y| |x⃗ − ⃗y′ |
and obviously the electric field in the sphere and induced charge on the inside of the sphere would
remain unchanged.
PHY 5346
HW Set 2 Solutions – Kimel

4. 2.1 We will work in cylidrical coordinate, (ρ, z, φ, with the charge q located at the point

d = dẑ , and the conducting plane is in the z = 0 plane.

Then we know from class the potential is given by

1 q q
φx⃗ = −
4π 0 x−⃗
⃗ d x+⃗
⃗ d

q ∂ 1 1
Ez = − −
4π 0 ∂z z − d 2 + ρ 2
1/2
z + d 2 + ρ 2
1/2

Ez =
q z−d − z+d
4π 0 3/2 3/2
z − d 2 + ρ 2 z + d 2 + ρ 2
a) σ =  0 E z z = 0

σ = 0
q −d − +d =−
q 1
4π 0 −d + ρ 22 3/2
+d + ρ 2
2 3/2 2πd 2 ρ 2
3
1 +
2
d
2

−1
Plotting 3
gives
1 2 +r 2 2

1 2 r 3 4 5
0

-0.2

-0.4

-0.6

-0.8

-1

b) Force of charge on plane


⃗= 1 q−q −ẑ  = 1 q2 ̂
F z
4π 0 2d 2 4 × 4π 0 d 2
c)
2

F = w = 0 E2 = σ2 = 1 −
q 1 = 1 q2
A 2 2 0 2 0 2πd 2 ρ 2
3
8 0
π2d4 1 + ρ2 3
12 + d
2
d2

q2 ∞ ρ q2 q2
F = 2π
8 0 π 2 d 4
∫0 ρ2 3
dρ = 2π
8 0 π 2 d 4
1 d2
4
= 1
4 × 4π 0 d 2
1+ d2
d)
∞ q2 ∞ q2
W= ∫ d Fdz = 4 × 4π 0
∫d dz =
z2 4 × 4π 0 d
e)
qiqj q2
W= 1 1
2 4π 0 > |x⃗i − ⃗
xj |
=−
2 × 4π 0 d
i,j,i≠j

Notice parts d) and e) are not equal in magnitude, because in d) the image moves when q moves.

f) 1 Angstrom = 10 −10 m, q = e = 1. 6 × 10 −19 C.

q2 −19
W= =e e = e 1. 6 × 10−10 9 × 10 9 V = 3.6 eV
4 × 4π 0 d 4 × 4π 0 d 4 × 10
PHY 5346
HW Set 2 Solutions – Kimel

3. 1.9 I will be using the principle of virtual work. In the figure below, Fδl is the work done by an
external force. If F is along δl (ie. is positive), then the force between the plates is attractive. This
work goes into increasing the electrostatic energy carried by the electric field and into forcing charge
into the battery holding the plates at constant potential φ 12 .

Conservation of energy gives


Fδl = δW + δQφ 12
or
∂Q
F = ∂W + φ 12
∂l ∂l
From problem 1.8,

a) Charge fixed.
1) Parallel plate capacitor
dQ 2
φ2 dQ
W = 1  0 A 12 , φ 12 = → W = 1  0 A A 0 = d Q2
2 d A 0 2 d 2 0 A
∂Q 2
= 0, F = ∂W =
Q
(attractive)
∂l ∂l 2 0A
2) Parallel cylinder capacitor
φ 12 = λ0 ln da , a = a 1 a 2
λQ
W = 1 Qφ 12 → F = ∂W = 1 Q λ0 ∂ ln da  = 1 (attractive)
2 ∂l 2 ∂d 2 0d
b) Potential fixed

1) Parallel plate capacitor


φ 12 A 0 ∂ Q = φ 12 A 0
Using Gauss’s law, Q = ,
d ∂l d2
Qd 2
φ2 φ 212 A 0 1  0 A φ 12 = 1  0 A
2
0A
→ F = − 1  0 A 12 + = = 1 Q2
2 d2 d2 2 d2 2 d 2 2 0 A
2) Parallel cylinder capacitor
 Lφ
W = 1 Qφ 12 , and Q = 0 d 12
2 ln a 
so
 Lφ 2 φ 212
W = 1 0 d 12 , ∂W = − 1  0 L
2 ln a  ∂l 2 ln 2 da d
∂Q φ 12
= 0L
∂l ln 2 da d
φ 212 φ 212 φ 212
F = − 1 0L +  0 L = 1 L
0
2 ln 2 da d ln 2 da d 2 ln 2 da d
PHY 5346
HW Set 2 Solutions – Kimel

⃗ ⋅ da
2. 1.8 We will be using Gauss’s law ∫ E ⃗= Q enc
0
a) 1) Parallel plate capacitor

σ Q φ 12 A 0 φ 12
From Gauss’s law E = 0 = A 0
= d
→Q= d
Q 2 A 0 φ 12 2
0 Ad φ
W = 0 ∫ E 2 d 3 x =  0 E Ad =
 2 Q2 2
A 0
= 1 d= 1 d
d = 1  0 A 12
2 2 2 2 0 A 2 0 A 2 d
2) Spherical capacitor

Q
From Gauss’s law, E = 1
4π 0 r 2
, a < r < b.

φ 12 = ∫ a Edr =
b Q
∫ a r −2 dr =
b
1 Q b − a → Q = 4π 0 baφ 12
4π 0 4 π 0 ba b − a
0 0 0 1 Q b − a
2 2 2
∫ E2d3x = ∫ a 4π r rdr
b
−4π −b + a
Q 2 Q
W= = =
2 2 4π 0 4 2 4π 0 ba 8 0 π ba
4π 0 baφ 12 2
b−a b − a φ 212
W= 1 π = 2π 0 ba
8 0 ba b − a
3) Cylindrical conductor
λL Q
From Gauss’ s law, E2πrL = 0
= 0

∫ a Edr = ∫a
b Q b Q
φ 12 = dr = ln ba
2π 0 L r 2π 0 L
0 0 2 2
∫ E2d3x = 2πL ∫ rdr
Q b
W= = 1 Q ln b
2 2 2π 0 L a r
2 4π 0 L a
2
2π 0 Lφ 12
1 ln b
φ 212
W= a
ln ba = π 0 L
4π 0 L ln ba
0
b) w = E
2
2

0 Q 2 1 Q
2
1) wr = 2 A 0
= 2 0 A 2
0 < r < d, = 0 otherwise.

0 Q 2 Q2
2) wr = 2
1
4π 0 r 2
= 1
32 0 π r 4
2 , a < r < b, = 0, otherwise

0 Q 2 Q2
3) wr = 2 2π 0 Lr
= 1
8 0 π 2 L 2 r 2
, = 0 otherwise.
PHY 5346
HW Set 1 Solutions – Kimel

4. 1.5
qe −αr 1 + αr2
φr⃗ =
4π 0 r
ρ
∇ 2 φr⃗ = −  0 , ∇ 2 = 12 ∂ r 2 ∂
r ∂r ∂r
−αr
∇2φ =
q 1 ∂ r2 ∂ e α −αr
4π 0 r 2 ∂r ∂r r + 2e
Using ∇ 2 1
r = −4πδr⃗
q 1 ∂ −αr −αr 2 ∂ 1 − α 2 r 2 e −αr
∇2φ = −αre + e r
4π 0 r 2 ∂r ∂r r 2
2 −αr −αr 2 −αr 3 −αr
− α2 e −αr + α er + αe 2 − 4πδr⃗ − α er + α e
q
=
4π 0 r r 2
3
qα −αr
= − 10 qδr⃗ − e

qα 3 −αr
ρr⃗ = qδr⃗ − e

That is, the charge distribution consists of a positive point charge at the origin, plus an
exponentially decreasing negatively charged cloud.
PHY 5346
HW Set 1 Solutions – Kimel

3. 1.4 Gauss’s Law:


∫ E⃗ ⋅ da⃗ =
Q enclosed
0
Q
a) Conducting sphere: all of the charge is on the surface σ = 4πa 2

Q
E4πr 2 = 0, r < a E4πr 2 =  0 , r > a

E = 0, r < a ⃗ =
E
Q ̂
r, r > a
4π 0 r 2

Q
b) Uniform charge density: ρ = 4 πa 3
, r < a, ρ = 0, r > a.
3

Qr 3 Qr
E4πr 2 = →E= , r<a
0a 3
4π 0 a 3
Q Q
E4πr 2 =  0 → E = , r>a
4π 0 r 2

c) ρ = A r n
n + 3Q n
Q = 4π ∫ r 2 drAr n = 4πAa n+3 /n + 3 → ρ = r ,r<a
4πa n+3
ρ = 0, r > a
n + 3Q Q r n+3
E4πr 2 = n+3
4πr n+3 /n + 3 → E = , r<a
4π 0 a 4π 0 r 2 a n+3
Q
E= , r>a
4π 0 r 2
1. n = −2.
Q
E= , r<a
4π 0 ra
Q
E= , r>a
4π 0 r 2

2. n = 2.
Qr 3
E= , r<a
4π 0 a 5
Q
E= , r>a
4π 0 r 2
PHY 5346
HW Set 1 Solutions – Kimel

2
1.3 In general we take the charge density to be of the form ρ = fr⃗δ, where fr⃗ is determined by
physical constraints, such as ∫ ρd 3 x = Q.
a) variables: r, θ, φ. d 3 x = dφd cos θr 2 dr
ρ = fr⃗δr − R = frδr − R = fRδr − R

∫ ρd 3 x = fR ∫ r 2 drdΩdr − R = 4πfRR 2 = Q → fR = Q


4πR 2
Q
ρr⃗ = δr − R
4πR 2
b) variables: r, φ, z. d 3 x = dφdzrdr
ρr⃗ = fr⃗δr − b = fbδr − b

∫ ρd 3 x = fb ∫ dzdφrdrδr − b = 2πfbbL = λL → fb = λ


2πb
ρr⃗ = λ δr − b
2πb
c) variables: r, φ, z. d 3 x = dφdzrdr Choose the center of the disk at the origin, and the z-axis
perpendicular to the plane of the disk
ρr⃗ = fr⃗δzθR − r = fδzθR − r
where θR − r is a step function.
∫ ρd 3 x = f ∫ δzθR − rdφdzrdr = 2π R2 f = Q → f = πRQ 2
2

Q
ρr⃗ = δzθR − r
πR 2
d) variables: r, θ, φ. d 3 x = dφd cos θr 2 dr
ρr⃗ = fr⃗δcos θθR − r = frδcos θθR − r

∫ ρd 3 x = ∫ frδcos θθR − rdφd cos θr 2 dr = ∫ 0 frrrdrdφ = 2πN ∫ 0 rdr = πR 2 N = Q


R R

where I’ve used the fact that rdrdφ is an element of area and that the charge density is uniformly
distributed over area.
Q
ρr⃗ = δcos θθR − r
πR 2 r

Das könnte Ihnen auch gefallen